Please remember that CME credit no longer is available for this course

Education Module Learner Questions January 04 Asessment Summary CME Credit Expired Page     1  2  3  4  5  6  7  8  9  10     Help  |  Table of C...
Author: Phebe Webster
2 downloads 1 Views 4MB Size
Education Module Learner

Questions

January 04

Asessment Summary

CME Credit Expired

Page     1  2  3  4  5  6  7  8  9  10     Help  |  Table of Contents

Overview

Please remember that CME credit no longer is available for this course.

Assessment 04 1

January 04

2

February 04

3

March 04

4

April 04

5

May 04

6

June 04

7

July 04

8

August 04

9

September 04

Numerous therapeutic agents are known to have teratogenic effects on the developing fetus. Of the following, the findings in the newborn that are MOST suggestive of maternal ingestion of an angiotensin converting enzyme inhibitor are deafness and cataracts microtia and conotruncal malformation nasal hypoplasia and stippled epiphyses neonatal anuria and patent ductus arteriosus smooth philtrum and lip

You selected

10 October 04

, the correct answer is

.

Angiotensin converting enzyme (ACE) inhibitors are used in the treatment of hypertension and the management of heart failure. Studies of this class of agents in pregnant animals have revealed an increased incidence of renal defects and fetal death. In humans, the use of ACE inhibitors throughout pregnancy or during the second and third trimesters has been associated with a number of complications, including fetal oligohydramnios and neonatal anuria due to persistent inhibition of the renin-angiotensin system as well as fetal hypotension with subsequent poor perfusion of tissues. Patent ductus arteriosus also has been reported and has been hypothesized to result from the effect of these agents on kinase II, which increases production of prostaglandin. Hypoplasia of the skull, spontaneous abortion, and intrauterine demise also have been reported. Thus, ACE inhibitors should not be used during pregnancy.

November 04 December 12 04 11

NeoReviews Basic Self Assessment Return to NeoReviews.org

Microtia and conotruncal malformations are associated with prenatal exposure to retinoic acid. Nasal hypoplasia and stippled epiphyses are characteristic of the fetal warfarin syndrome, which results from first trimester exposure to coumarin derivatives. Deafness and cataracts are associated with rubella infections during the first trimester of pregnancy. Smooth philtrum and a thin upper lip are part of the facial characteristics of infants who have fetal alcohol syndrome. References: Bhatt-Mehta V, Deluga KS. Fetal exposure to lisinopril: neonatal manifestations and management. Pharmacotherapy. 1993;13:515-518 Kreft-Jais C, Plouin PF, Tchobroutsky C, Boutroy MJ. Angiotensin-converting enzyme inhibitors during pregnancy: a survey of 22 patients given captopril and nine given enalapril. Br J Obstet Gynaecol. 1988;95: 420-422 Content Specification(s): Know the effects on the fetus of acute and chronic renal disease and their treatment Know the effects on the fetus of maternal chronic hypertension and its treatment

 

 

http://emb.aap.org/courseprodv2/Index.asp[3/29/2012 11:56:05 AM]

 

  

Education Module Learner

http://emb.aap.org/courseprodv2/Index.asp[3/29/2012 11:56:05 AM]

Education Module Learner

Questions

January 04

Asessment Summary

CME Credit Expired

Page     1  2  3  4  5  6  7  8  9  10     Help  |  Table of Contents

Overview Assessment 04 1

January 04

2

February 04

3

March 04

4

April 04

5

May 04

6

June 04

7

July 04

8

August 04

9

September 04

10 October 04 November 04 December 12 04 11

NeoReviews Basic Self Assessment Return to NeoReviews.org

A newborn has ambiguous genitalia characterized by posterior fusion of the labioscrotal folds and clitoromegaly. Findings include: chromosome analysis, a normal female 46,XX pattern; sodium, 127 mEq/L; potassium, 6.5 mEq/L; markedly increased levels of 17-hydroxyprogesterone and androstenedione. You diagnose the salt-wasting form of congenital adrenal hyperplasia (CAH) due to 21-hydroxylase deficiency. When discussing options for future pregnancies, the MOST appropriate statement to include is that CAH is identified best by measurement of amniotic fluid 17-hydroxyprogesterone prenatal diagnosis of CAH can be determined by molecular analysis of fetal DNA the fetal gender should be determined by ultrasonography because only females are at risk the mother should receive dexamethasone therapy throughout all future pregnancies there is a 50% risk for an affected child in each future pregnancy

You selected

, the correct answer is

.

Congenital adrenal hyperplasia (CAH) is an autosomal recessive disorder that results from deficiency of the enzyme 21-hydroxylase. The gene for this enzyme has been identified, and it is now possible to determine the exact molecular defect(s) in most families, which permits precise prenatal diagnosis by examination of fetal DNA obtained by chorionic villus sampling or amniocentesis. To facilitate such studies, the affected child and both parents should undergo genetic counseling and molecular genotyping of the 21-hydroxylase gene prior to initiating another pregnancy. In addition, couples at risk should be advised that early detection in all future pregnancies is necessary to prevent or minimize the virilizing effects on affected female fetuses. CAH occurs as two distinct phenotypes: a simple virilizing form (25% to 33% of patients) and a salt-wasting form (67% to 75% of patients). In both forms, the deficiency of 21-hydroxylase results in decreased cortisol synthesis by the adrenal gland and the increased production of cortisol precursors (eg, 17hydroxyprogesterone) and androgens, which do not require 21-hydroxylase for their synthesis. Because fetal genital development is regulated by adrenal steroid synthesis, the presence of elevated androgen levels in affected female fetuses results in variable degrees of virilization. In the salt-wasting form of the disease, renal salt wasting due to deficient aldosterone synthesis also occurs. The degree of virilization in affected females is not predictive of salt wasting, and even those who have mild virilization must be monitored for this possibility. However, the presence or absence of salt wasting usually is consistent in affected members of the same family. Thus, for the family described in the clinical vignette, any future affected infants also would be expected to have the salt-wasting form of CAH. The management goals for pregnancies at risk for CAH are to identify affected fetuses of both sexes and to minimize virilization in affected females. To achieve these goals, oral dexamethasone therapy must be initiated as soon as the pregnancy is detected. This semisynthetic steroid can cross the placenta and suppress the fetal adrenal gland in affected female fetuses, thereby minimizing the virilizing effects. Fetal cells then are obtained either by chorionic villus sampling at 10 weeks'

http://emb.aap.org/courseprodv2/Index.asp[3/29/2012 11:57:53 AM]

Education Module Learner

gestation or by amniocentesis at 15 to 18 weeks' gestation. DNA from these cells then is used to determine the fetal sex by chromosome analysis and for molecular testing (ie, 21-hydroxylase genotyping) to determine the disease status. If a male fetus is identified by the chromosome analysis, the dexamethasone can be discontinued. If the fetus is identified as female, the dexamethasone is discontinued only if results of the molecular analysis reveal that the fetus is unaffected. For pregnancies involving affected female fetuses, treatment is continued until delivery. All infants should be tested at birth to confirm the prenatal diagnosis by measuring 17-hydroxyprogesterone levels and repeating molecular analysis. Infants who were identified as affected by the prenatal testing also should have electrolyte levels measured on the first and second day of life to monitor for the salt-wasting form of CAH. Prior to the availability of molecular testing, determination of 17hydroxyprogesterone levels in amniotic fluid was used to identify fetuses who had CAH. Although 17-hydroxyprogesterone levels clearly are elevated in the severe saltwasting form of CAH, they may be normal in the simple virilizing form. Thus, this method lacks sensitivity and has been replaced by molecular testing for those families in which the mutation has been identified. Although ultrasonographic determination of fetal sex is possible, the prenatal management of CAH requires precise determination of the sex of the fetus, which is achieved best by fetal cell sampling and chromosome analysis. Because CAH is inherited as an autosomal recessive trait, the risk for an affected infant in each pregnancy is 25%. References: Karaviti LP, Mercado AB, Mercado MB, et al. Prenatal diagnosis/treatment in families at risk for infants with steroid 21-hydroxylase deficiency (congenital adrenal hyperplasia). J Steroid Biochem Mol Biol. 1992;41: 445-451 New MI, Crawford C, Wilson RC. Genetic disorders of the adrenal gland. In: Rimoin DL, Connor JM, Pyeritz RE, eds. Emery and Rimoin's Principles and Practice of Medical Genetics. 3rd ed. New York, NY: Churchill Livingstone; 1997:1441-1476 Content Specification(s) Know the etiology and diagnosis of an infant with ambiguous genitalia, including congenital adrenal hyperplasia

 

 

http://emb.aap.org/courseprodv2/Index.asp[3/29/2012 11:57:53 AM]

 

  

Education Module Learner

Questions

January 04

Asessment Summary

CME Credit Expired

Page     1  2  3  4  5  6  7  8  9  10     Help  |  Table of Contents

Overview

1

January 04

A 1-day-old infant appears dusky during feeding. Oxygen is administered via nasal cannula, and 2 hours later she develops tachypnea. Findings include: heart rate, 170 beats/min; respiratory rate, 80 breaths/min; right arm blood pressure, 48/30 mm Hg; right leg blood pressure, 52/32 mm Hg; and pulse oximetry, 90% on oxygen. You suspect hypoplastic left heart syndrome.

2

February 04

Of the following, the MOST likely findings include

3

March 04

4

April 04

5

May 04

6

June 04

7

July 04

8

August 04

9

September 04

Assessment 04

10 October 04 November 04 December 12 04 11

NeoReviews Basic Self Assessment Return to NeoReviews.org

continuous ductal murmur, bounding pulses continuous ductal murmur, poor peripheral pulses holosystolic murmur, poor peripheral pulses, quiet second heart sound no murmur, precordial hyperactivity, loud second heart sound no murmur, precordial hyperactivity, quiet second heart sound

You selected

, the correct answer is

.

Infants who have hypoplastic left heart (HLH) syndrome develop signs of shock, including poor pulses and metabolic acidosis, as systemic perfusion deteriorates. Ductal closure in these infants results in inadequate blood flow to the body from the functional single ventricle because the ductus arteriosus is the only path for blood to flow from the right ventricle to the body. Even if the ductus remains open, a drop in pulmonary vascular resistance "steals" blood to the pulmonary circulation, thereby depriving the systemic circulation of adequate perfusion. For this reason, mildly cyanotic infants who have HLH syndrome often deteriorate suddenly after oxygen is administered, because the oxygen acts as a pulmonary vasodilator. The most obvious physical findings in newborns who have HLH syndrome are a hyperdynamic precordium and a loud, even palpable single second heart sound (S2). The hyperactive precordium is due to the greatly enlarged right ventricle that is contracting against systemic pressure. The strikingly loud S2 is caused by the pulmonary artery functioning as a de facto aorta, pumping blood to the body through the ductus arteriosus. Therefore, the pulmonic closure sound is loud as a result of the high end systolic pressure found in the large pulmonary artery located just beneath the chest wall to the left of the sternum. A continuous murmur is not heard in infants who have HLH syndrome, even though the ductus arteriosus remains open. A murmur from a ductus is created by highvelocity flow from a higher pressure aorta into a lower pressure pulmonary artery. In patients who have HLH, the pulmonary artery pressure is equal to or exceeds that of the aorta. Because flow from the systemic right ventricle to this large hypertensive pulmonary artery is not obstructed or turbulent, there is no reason for a significant murmur to occur. A holosystolic murmur will be present only if there is very significant regurgitation of the tricuspid valve, which is unusual in affected infants. Diminished peripheral pulses are a sign of HLH syndrome, particularly as the ductus starts to close or as pulmonary vascular resistance falls. Both of these changes diminish the flow of systemic blood from the main pulmonary artery through the ductus to the systemic circulation. Maintaining ductal patency by administering an infusion of prostaglandin E1 often is not sufficient to provide adequate systemic blood flow in infants who have HLH

http://emb.aap.org/courseprodv2/Index.asp[3/29/2012 11:58:10 AM]

Education Module Learner

syndrome. Use of the so-called "chemical banding" of the pulmonary vasculature employs ventilatory strategies to increase pulmonary vascular resistance. Administration of room air while on the ventilator (or even Fio2 less than 21% using carbon dioxide in the ventilator circuit) and muscle relaxant drugs to prevent spontaneous hyperventilation with secondary pulmonary vasodilation often can reverse metabolic acidosis and re-establish good renal and systemic perfusion after prostaglandin E1 infusion has opened the ductus arteriosus. References: Victorica BE. Newborns with low systemic output. In: Gessner IH, Victorica BE, eds. Pediatric Cardiology: A Problem Oriented Approach. Philadelphia, Pa: WB Saunders Co; 1993:111-115 Wernovsky G, Bove EL. Single ventricle lesions: excessive pulmonary blood flow (congestive heart failure). In: Chang AC, Hanley FL, Wernovsky G, Wessel DL, eds. Pediatric Cardiac Intensive Care. Baltimore, Md: Williams & Wilkins; 1998:274  Content Specification(s): Recognize the clinical features of a neonate with a left-sided cardiac obstructive lesion

 

 

http://emb.aap.org/courseprodv2/Index.asp[3/29/2012 11:58:10 AM]

 

  

Education Module Learner

Questions

January 04

Asessment Summary

CME Credit Expired

Page     1  2  3  4  5  6  7  8  9  10     Help  |  Table of Contents

Overview

1

January 04

An 8-week-old child is being seen for a health supervision visit by his pediatrician. His mother reports that he is wheezing and having increasing difficulty while feeding. Physical examination reveals a raised, erythematous birthmark on the chest and biphasic stridor on auscultation.  The pediatrician calls a neonatologist to discuss the case.

2

February 04

Of the following, the MOST likely finding would be

3

March 04

4

April 04

5

May 04

6

June 04

7

July 04

obstructing adenoids on lateral neck radiography

8

August 04

unilateral air trapping on chest radiography

9

September 04

Assessment 04

10 October 04 November 04 December 12 04 11

NeoReviews Basic Self Assessment Return to NeoReviews.org

a prolapsing epiglottis on nasopharyngoscopy a unilateral subglottic mass on fluoroscopy left mainstem bronchomalacia on bronchoscopy

You selected

, the correct answer is

.

Biphasic stridor is produced by rapid turbulent flow of air through a narrowed segment of the airway. Stridor that is biphasic suggests an anatomic location at the glottic or subglottic level. Inspiratory stridor typically is produced by an obstructive lesion above the vocal cords, and expiratory stridor indicates an intrathoracic site of obstruction. Subglottic hemangioma is a benign vascular malformation that typically enlarges over the first few months of life. Patients often are asymptomatic or minimally symptomatic at birth, then develop biphasic stridor, as reported for the infant in the vignette. The natural history is gradual enlargement until 1 year of age followed by slow involution to complete resolution. Fluoroscopy most commonly reveals a unilateral subglottic mass. Fifty percent of affected children have an associated cutaneous hemangioma, such as the erythematous birthmark described in the vignette. Definitive diagnosis is made by microlaryngoscopy in the operating room. Carbon dioxide laser excision, systemic or intralesional steroids, and tracheotomy have been used for treatment. Inspiratory stridor in infancy most commonly is caused by laryngomalacia or prolapse of the supraglottic structures into the glottic introitus upon inspiration. Laryngomalacia can be diagnosed by nasopharyngoscopy. It usually is benign and resolves without intervention in 90% of patients by 1-1/2 years. Vocal cord paralysis is another common cause of inspiratory stridor that may have a mild expiratory component. Airway fluoroscopy or nasopharyngoscopy is diagnostic. Expiratory stridor most commonly is caused by tracheomalacia. This may present at birth and improve slowly over time; intervention rarely is required. Vascular compression may occur from an aberrant innominate artery that can compress the anterior trachea. Vascular rings (eg, double aortic arch) can compress both the trachea and esophagus. Retroesophageal subclavian compression typically causes dysphagia, but it also may result in respiratory symptoms. A diagnosis of tracheal anomaly and vascular lesion may be suggested by findings on chest radiography or airway fluoroscopy. Barium swallow demonstrates a typical indentation from posterior vascular compression in patients who have vascular rings or an aberrant subclavian artery, but it will not identify anterior compression from an aberrant

http://emb.aap.org/courseprodv2/Index.asp[3/29/2012 11:58:29 AM]

Education Module Learner

innominate artery. Flexible or rigid bronchoscopy can be used to confirm the degree of compression of the trachea from any cause. In patients who have suspected vascular airway compression, magnetic resonance imaging or computed tomography is used to define abnormal vascular anatomy. Obstructing adenoids can cause upper airway obstruction. Patients typically snore without accompanying stridor when tonsils and adenoids are hypertrophied. Although lateral neck radiography is a helpful diagnostic test for children who have adenoid hypertrophy, the clinical findings described in the vignette do not make this diagnosis likely. Congenital subglottic stenosis may be either membranous or cartilaginous. Stridor is typically present at birth and may worsen slightly over time. The diagnosis may be made by plain radiography, fluoroscopy, or endoscopy. Microlaryngoscopy in the operating room is the most accurate diagnostic method. The anomaly usually is symmetric and bilateral or circumferential. Unilateral air trapping on chest radiography is suggestive of unilateral compression or obstruction of the bronchus. In older infants, the most common cause is a foreign body. Bronchomalacia or other bronchial anomalies are diagnosed most commonly by bronchoscopy, but clinical findings would include expiratory wheezing rather than biphasic stridor. References: Brodsky L. Congenital stridor. Pediatr Rev. 1996;17: 408-411 Cotton RT, Reilly JT. Stridor and airway obstruction. In: Bluestone CD, Stool SE, Kenna MA, eds. Pediatric Otolaryngology. 3rd ed. Philadelphia, Pa: WB Saunders Co; 1996:1275-1287 Gregg CM, Wiatrak BJ, Koopmann CF Jr. Management options for infantile subglottic hemangioma. Am J Otolaryngol. 1995;16:409-414 Holinger LD. Etiology of stridor in the neonate, infant and child. Ann Otol Laryngol. 1980;89:397-400 Content specification(s): Know the various causes of stridor in the newborn

 

 

http://emb.aap.org/courseprodv2/Index.asp[3/29/2012 11:58:29 AM]

 

  

Education Module Learner

Questions

January 04

Asessment Summary

CME Credit Expired

Page     1  2  3  4  5  6  7  8  9  10     Help  |  Table of Contents

Overview Assessment 04

A 2-week-old neonate who was born at 32 weeks' gestation has recovered from respiratory distress syndrome. He has been tolerating increasing volumes of enteral feedings via gavage. Over the past several feedings, abdominal distension, gastric residuals, and stools that are positive for blood have been noted.

1

January 04

2

February 04

3

March 04

4

April 04

5

May 04

6

June 04

7

July 04

pneumatosis intestinalis

8

August 04

thickening of the bowel wall

9

September 04

10 October 04 November 04 December 12 04 11

NeoReviews Basic Self Assessment Return to NeoReviews.org

Of the following, the radiographic finding MOST supportive of the diagnosis of necrotizing enterocolitis is: absence of luminal bowel gas generalized bowel distension intraperitoneal fluid

You selected

, the correct answer is

.

Pneumatosis intestinalis (gas in the bowel wall) is the radiographic hallmark of necrotizing enterocolitis (NEC) and is used to confirm the diagnosis suggested by clinical symptoms and signs in the high-risk neonate described in the vignette. Pneumatosis intestinalis may be seen as a bubbly or foamy gas pattern or as a linear or crescent distribution of gas in the bowel wall. It may extend into the portal venous circulation and be visible as linear branching lucencies overlying the liver. Portal venous gas is associated with severe disease. The pattern of luminal bowel gas in infants who have NEC is variable and nonspecific. It is decreased in some, but this finding also can be seen in the absence of NEC in sick infants, particularly those receiving drugs for muscle paralysis during mechanical ventilation. Generalized bowel distension is an early radiographic sign of NEC in some infants, but it also is seen in infants who are receiving nasal continuous positive airway pressure. A single persistent dilated loop of bowel rather than generalized bowel distension is more characteristic of NEC. Multiple dilated loops of bowel, particularly when localized in the right lower quadrant, also are consistent with NEC. When intraperitoneal fluid is present, the gas-filled loops of bowel float centrally in the abdomen and are separated from each other in the fluid. Ultrasonography is more sensitive and specific than radiography for detecting intraperitoneal fluid. Fluid also may accumulate in the peritoneal cavity as a result of other from causes (eg, hypoalbuminemia). In infants who have NEC, loops of bowel may be separated from each other by bowel walls that have been thickened by edema and hemorrhage. However, detection of thickening of the bowel wall requires a somewhat subjective interpretation and is nonspecific for the diagnosis of NEC. NEC affects predominantly preterm infants, with a markedly increased incidence in infants at younger gestational ages. Abdominal distension and gastric residuals, such as described for the infant in the vignette, are signs of feeding intolerance. The presence of bloody stools suggests that there has been necrosis of intestinal epithelium. A prudent management approach would be to stop enteral feeding, initiate antibiotic therapy, and observe for symptoms (eg, lethargy, apnea, poor

http://emb.aap.org/courseprodv2/Index.asp[3/29/2012 11:58:46 AM]

Education Module Learner

perfusion) and signs (eg, abdominal tenderness, muscular guarding, erythema of abdominal wall) that would indicate more serious involvement. Serial laboratory tests are important to evaluate the progression of the disease. Leukopenia, leukocytosis, thrombocytopenia, anemia, coagulopathy, electrolyte imbalance, acidosis, hypoxia, or hypercapnia are indicative of worsening disease. Serial abdominal radiographs help to determine the need for surgical intervention. Evidence of intestinal perforation or worsening disease despite appropriate medical management or are indications for surgery in infants who have NEC. References: Crissinger KD. Necrotizing enterocolitis. In: Fanaroff AA, Martin RJ, eds. NeonatalPerinatal Medicine: Diseases of the Fetus and Infant. 6th ed. St Louis, Mo: Mosby-Year Book, Inc; 1997:1333-1337 Morrison SC, Jacobson JM. The radiology of necrotizing enterocolitis. Clin Perinatol. 1994;21:347-363. Content Specification(s):  Understand the clinical manifestations, laboratory and radiographic findings of NEC.

 

 

http://emb.aap.org/courseprodv2/Index.asp[3/29/2012 11:58:46 AM]

 

  

Education Module Learner

Questions

January 04

Asessment Summary

CME Credit Expired

Page     1  2  3  4  5  6  7  8  9  10     Help  |  Table of Contents

Overview Assessment 04 1

January 04

2

February 04

3

March 04

4

April 04

5

May 04

6

June 04

7

July 04

8

August 04

9

September 04

10 October 04 November 04 December 12 04 11

NeoReviews Basic Self Assessment Return to NeoReviews.org

You are examining a newborn in the nursery and palpate a large mass in the abdomen. Of the following, the MOST likely diagnosis is:

autosomal dominant polycystic kidney disease horseshoe kidney multicystic kidney dysplasia renal vein thrombosis Wilms tumor

You selected

, the correct answer is

.

A palpable abdominal mass in a newborn constitutes a medical emergency. The most common cause of an abdominal mass in the neonatal period is hydronephrosis. One common cause of hydronephrosis is multicystic kidney dysplasia (MKD). This congenital malformation is caused by obstruction to urine flow during in utero development, which results in abnormal parenchymal development. Renal ultrasonography reveals multiple cysts scattered throughout the renal parenchyma, with marked echogenicity of the parenchyma, indicating nonspecific renal dysplasia. The cysts are of varying size, often replacing normal renal tissue. Because the anomaly usually is unilateral, most children who have MKD have normal renal function. However, because at least 50% of patients have an abnormality of the contralateral kidney, voiding cystourethrography (VCUG) to evaluate for vesicoureteral reflux is essential. Other anomalies of the contralateral kidney include ureteropelvic junction obstruction (with hydronephrosis), duplicated ureters, renal dysplasia, and hydronephrosis. If MKD is bilateral, renal failure is assured. MKD is associated with other malformations, including CHARGE syndrome; Jeune syndrome; fetal alcohol syndrome; Marfan syndrome; Noonan syndrome; prune belly (Eagle-Barrett) syndrome; rubella syndrome; trisomies 8, 9, 13, 18, 21, and 22; tuberous sclerosis; and Zellweger syndrome. Additionally, MKD occurs in infants of diabetic mothers. Serum electrolyte concentrations must be obtained in any newborn who has MKD. In most instances, it is difficult to distinguish between hydronephrosis and MKD. Therefore, a renal scan (eg, DMSA scan) should be performed. If the patient has hydronephrosis, evidence of renal function will be apparent on the scan. However, function may not be demonstrated with MKD. If the contralateral kidney is large and the serum creatinine level is normal, the contralateral kidney probably is normal, but follow-up renal ultrasonography is required to demonstrate progressive function of the contralateral kidney and further atrophy of the affected kidney. Repeat ultrasonography and measurement of serum creatinine levels annually are appropriate follow-up. Treatment neither is required nor available for unilateral MKD. Most affected kidneys eventually atrophy; hence, prior recommendations to perform nephrectomy in anticipation of possible malignant transformation no longer are advocated. However, annual renal ultrasonography should be performed to assess the size of the affected kidney.

http://emb.aap.org/courseprodv2/Index.asp[3/29/2012 11:59:02 AM]

Education Module Learner

Autosomal dominant polycystic kidney disease (ADPKD) is a very common cause of renal failure in adults. However, because it is a very slowly progressive disease, it rarely is seen in newborns. ADPKD typically presents with hematuria, urinary tract infection, and hypertension. Although horseshoe kidneys are infrequently observed in newborns, they may present with urinary tract infections, hematuria, or abdominal mass. Nephroblastoma is a general term used to describe a tumor of the kidney. The most common cause in early childhood is Wilms tumor, which presents as an abdominal or flank mass and may be heralded by hematuria and rarely, hypertension. It can be associated with other congenital anomalies, including genitourinary anomalies, hemihypertrophy, aniridia, gastrointestinal anomalies, polydactyly, and hydrocephalus. Although this is the most common solid tumor in childhood, it rarely presents during the newborn period. The typical presentation of renal vein thrombosis in a child is gross hematuria and an abdominal mass. Although rare, it often results in complete loss of function of the affected kidney. References: Atiyeh B, Husmann D, Baum M. Contralateral renal abnormalities in multicysticdysplastic kidney disease. J Pediatr. 1992;121:65-67 Kleiner B, Filly RA, Mack L, Callen PW. Multicystic dysplastic kidney: observations of contralateral disease in the fetal population. Radiology. 1986;161:27-29 Pathak IG, Williams DI. Multicystic and cystic dysplastic kidneys. Br J Urol. 1964;36:318-331 Content Specification(s): Recognize the clinical and laboratory features of common neonatal malignancies, including teratomas, hemangiomas, neuroblastoma, Wilms tumor, retinoblastoma.

 

 

http://emb.aap.org/courseprodv2/Index.asp[3/29/2012 11:59:02 AM]

 

  

Education Module Learner

Questions

January 04

Asessment Summary

CME Credit Expired

Page     1  2  3  4  5  6  7  8  9  10     Help  |  Table of Contents

Overview Assessment 04 1

January 04

2

February 04

3

March 04

4

April 04

5

May 04

6

June 04

7

July 04

8

August 04

9

September 04

10 October 04 November 04 December 12 04 11

NeoReviews Basic Self Assessment Return to NeoReviews.org

You are examining a newborn in the nursery and palpate a large mass in the abdomen. Of the following, the MOST likely diagnosis is:

autosomal dominant polycystic kidney disease horseshoe kidney multicystic kidney dysplasia renal vein thrombosis Wilms tumor

You selected

, the correct answer is

.

A palpable abdominal mass in a newborn constitutes a medical emergency. The most common cause of an abdominal mass in the neonatal period is hydronephrosis. One common cause of hydronephrosis is multicystic kidney dysplasia (MKD). This congenital malformation is caused by obstruction to urine flow during in utero development, which results in abnormal parenchymal development. Renal ultrasonography reveals multiple cysts scattered throughout the renal parenchyma, with marked echogenicity of the parenchyma, indicating nonspecific renal dysplasia. The cysts are of varying size, often replacing normal renal tissue. Because the anomaly usually is unilateral, most children who have MKD have normal renal function. However, because at least 50% of patients have an abnormality of the contralateral kidney, voiding cystourethrography (VCUG) to evaluate for vesicoureteral reflux is essential. Other anomalies of the contralateral kidney include ureteropelvic junction obstruction (with hydronephrosis), duplicated ureters, renal dysplasia, and hydronephrosis. If MKD is bilateral, renal failure is assured. MKD is associated with other malformations, including CHARGE syndrome; Jeune syndrome; fetal alcohol syndrome; Marfan syndrome; Noonan syndrome; prune belly (Eagle-Barrett) syndrome; rubella syndrome; trisomies 8, 9, 13, 18, 21, and 22; tuberous sclerosis; and Zellweger syndrome. Additionally, MKD occurs in infants of diabetic mothers. Serum electrolyte concentrations must be obtained in any newborn who has MKD. In most instances, it is difficult to distinguish between hydronephrosis and MKD. Therefore, a renal scan (eg, DMSA scan) should be performed. If the patient has hydronephrosis, evidence of renal function will be apparent on the scan. However, function may not be demonstrated with MKD. If the contralateral kidney is large and the serum creatinine level is normal, the contralateral kidney probably is normal, but follow-up renal ultrasonography is required to demonstrate progressive function of the contralateral kidney and further atrophy of the affected kidney. Repeat ultrasonography and measurement of serum creatinine levels annually are appropriate follow-up. Treatment neither is required nor available for unilateral MKD. Most affected kidneys eventually atrophy; hence, prior recommendations to perform nephrectomy in anticipation of possible malignant transformation no longer are advocated. However, annual renal ultrasonography should be performed to assess the size of the affected kidney.

http://emb.aap.org/courseprodv2/Index.asp[3/29/2012 11:59:19 AM]

Education Module Learner

Autosomal dominant polycystic kidney disease (ADPKD) is a very common cause of renal failure in adults. However, because it is a very slowly progressive disease, it rarely is seen in newborns. ADPKD typically presents with hematuria, urinary tract infection, and hypertension. Although horseshoe kidneys are infrequently observed in newborns, they may present with urinary tract infections, hematuria, or abdominal mass. Nephroblastoma is a general term used to describe a tumor of the kidney. The most common cause in early childhood is Wilms tumor, which presents as an abdominal or flank mass and may be heralded by hematuria and rarely, hypertension. It can be associated with other congenital anomalies, including genitourinary anomalies, hemihypertrophy, aniridia, gastrointestinal anomalies, polydactyly, and hydrocephalus. Although this is the most common solid tumor in childhood, it rarely presents during the newborn period. The typical presentation of renal vein thrombosis in a child is gross hematuria and an abdominal mass. Although rare, it often results in complete loss of function of the affected kidney. References: Atiyeh B, Husmann D, Baum M. Contralateral renal abnormalities in multicysticdysplastic kidney disease. J Pediatr. 1992;121:65-67 Kleiner B, Filly RA, Mack L, Callen PW. Multicystic dysplastic kidney: observations of contralateral disease in the fetal population. Radiology. 1986;161:27-29 Pathak IG, Williams DI. Multicystic and cystic dysplastic kidneys. Br J Urol. 1964;36:318-331 Content Specification(s): Recognize the clinical and laboratory features of common neonatal malignancies, including teratomas, hemangiomas, neuroblastoma, Wilms tumor, retinoblastoma.

 

 

http://emb.aap.org/courseprodv2/Index.asp[3/29/2012 11:59:19 AM]

 

  

Education Module Learner

Questions

January 04

Asessment Summary

CME Credit Expired

Page     1  2  3  4  5  6  7  8  9  10     Help  |  Table of Contents

Overview Assessment 04

At 2 days of age, a first-born male Asian term infant appears icteric. He is being breastfed; maternal lactation consists primarily of colostrum. The infant has voided once or twice every 8 hours and has had several meconium stools. His weight today is 6 oz (170 g) lower than at birth. Total serum bilirubin is 14 mg/dL (direct, 0.5 mg/dL) and hematocrit is 54%.

1

January 04

2

February 04

3

March 04

4

April 04

alpha-1-antitrypsin deficiency

5

May 04

Gilbert syndrome

6

June 04

7

July 04

8

August 04

hypothyroidism

9

September 04

physiologic jaundice

Of the following, the MOST likely contributing factor to this infant's elevated bilirubin is:

glucose-6-phosphate dehydrogenase deficiency

10 October 04

You selected

November 04 December 12 04

In the term newborn, physiologic jaundice is characterized by a progressive increase in the serum concentration of unconjugated bilirubin from approximately 2 mg/dL in cord blood to a peak of 5 to 14 mg/dL in early postnatal life. The rate of increase varies, depending on ethnicity. In Caucasian and African-American infants, the peak concentration of 5 to 6 mg/dL is reached between 60 and 72 hours after birth. In Asian infants, the peak concentration of 10 to 14 mg/dL is reached between 72 and 120 hours after birth. The infant described in the vignette has a serum bilirubin concentration-both unconjugated and conjugated fractions-that fall within this expected range.  Mild dehydration, as suggested by the weight loss, may be present. The passage of meconium stools argues against gastrointestinal obstruction, and the resultant enterohepatic circulation of unconjugated bilirubin should be intact. The normal hematocrit gives no indication of hemolysis or anemia.

11

NeoReviews Basic Self Assessment Return to NeoReviews.org

, the correct answer is

.

The normal pattern of physiologic jaundice is exaggerated or prolonged in specific disorders. These disorders can be classified according to the specific step in the pathway of bilirubin metabolism that is affected: bilirubin production, hepatic uptake, hepatic conjugation, or bilirubin excretion. The specific abnormality also determines whether the infant has unconjugated, conjugated, or mixed hyperbilirubinemia. The infant described in the vignette has unconjugated hyperbilirubinemia. Alpha-1-antitrypsin is a protease inhibitor of trypsin and several other enzymes. Deficiency of this inhibitor is inherited as an autosomal recessive trait and rarely may present in the first 8 weeks after birth with cholestasis and resultant conjugated hyperbilirubinemia. Progressive liver failure and death from cirrhosis are common by late childhood or early adulthood. Lung disease is prevalent in children who have a specific phenotype of alpha-1-antitrypsin deficiency. Approximately 10% of infants who have congenital hypothyroidism develop prolonged, exaggerated jaundice. Thyroid hormone affects many maturational processes, including hepatic bilirubin uptake and conjugation. Persistent unconjugated hyperbilirubinemia, therefore, is a frequent presenting symptom. Other manifestations of congenital hypothyroidism include lethargy, hypotonia, large

http://emb.aap.org/courseprodv2/Index.asp[3/29/2012 11:59:38 AM]

Education Module Learner

anterior and posterior fontanelles, feeding difficulty, respiratory distress, pallor, perioral cyanosis, mottled skin, poor or hoarse cry, constipation, and hypothermia, none of which is present in the infant described in the vignette. Some infants who have congenital hypothyroidism may be asymptomatic and be identified by neonatal screening for thyroid function. Treatment with thyroid hormone promptly resolves the hyperbilirubinemia. Glucose-6-phosphate dehydrogenase (G6PD) is an enzyme that catalyses the pentose phosphate pathway of glucose metabolism in the red blood cell. G6PD deficiency is an X-linked disorder that affects mostly males. Populations who have a high frequency of G6PD deficiency include African-Americans, southern Europeans, Sephardic Jews, Chinese, southeast Asians, and Arabs. The most common manifestation of G6PD deficiency is hemolysis that develops after being exposed to antioxidants (eg, naphthalene, primaquine, sulfonamides). Unconjugated hyperbilirubinemia, anemia, reticulocytosis, and morphologic changes in red blood cells (including Heinz bodies) are typical. The diagnosis of G6PD deficiency requires measurement of the level of enzyme activity in the red blood cell. Gilbert syndrome is an autosomal dominant disorder characterized by nonhemolytic unconjugated hyperbilirubinemia that results from a partial defect of bilirubin conjugation. Typically, the disease does not manifest in the neonatal period, and the hyperbilirubinemia is mild. Treatment with phenobarbital usually corrects the hyperbilirubinemia, presumably by enhancing hepatic bilirubin conjugation. References: Avery ME, Als H, Coulter DM. Jaundice. In: Avery ME, First LR, eds. Pediatric Medicine. 2nd ed. Baltimore, Md: Williams & Wilkins; 1994:223-233 Halamek LP, Stevenson DK. Conjugated hyperbilirubinemia. In: Fanaroff AA, Martin RJ, eds. Neonatal-Perinatal Medicine: Diseases of the Fetus and Infant. 6th ed. St Louis, Mo: Mosby-Year Book, Inc; 1997:1372-1383 Halamek LP, Stevenson DK. Neonatal jaundice and liver disease. In: Fanaroff AA, Martin RJ, eds. Neonatal-Perinatal Medicine: Diseases of the Fetus and Infant. 6th ed. St Louis, Mo: Mosby-Year Book, Inc; 1997:1345-1389 Provisional Committee on Quality Improvement and Subcommittee on Hyperbilirubinemia. Practice parameter: Management of hyperbilirubinemia in the healthy term newborn. Pediatrics. 1994;94:558-565 Content Specification(s): Understand the differences between physiologic and nonphysiologic jaundice.

 

 

http://emb.aap.org/courseprodv2/Index.asp[3/29/2012 11:59:38 AM]

 

  

Education Module Learner

Questions

January 04

Asessment Summary

CME Credit Expired

Page     1  2  3  4  5  6  7  8  9  10     Help  |  Table of Contents

Overview Assessment 04 1

January 04

2

February 04

3

March 04

4

April 04

5

May 04

6

June 04

7

July 04

8

August 04

9

September 04

10 October 04 November 04 December 12 04 11

NeoReviews Basic Self Assessment Return to NeoReviews.org

During the discharge examination from the newborn nursery, a term female infant is noted to have bilateral swollen labia majora with slight hyperpigmentation and rugae. No masses are palpated in the labioscrotal folds. The clitoris appears normalsized, and the vaginal opening and urethra are easily visualized. Of the following, the MOST likely cause for these physical findings is maternal exposure to: androgens estrogens marijuana medroxyprogesterone

thyroid hormone You selected

, the correct answer is

.

Genital development can be influenced by a number of factors, such as the production of endogenous hormones, exposure to an exogenous source of hormones, a chromosomal abnormality, and the presence of an inherited enzymatic defect. The effect of hormones on the appearance of genitalia depends on the timing and duration of prenatal exposure. For example, androgenic drugs can cause masculinization or virilization of external female genitalia. In contrast, chromosomal abnormalities can affect the differentiation of internal sexual organs and lead to the development of ambiguous genitalia. Prior to 6 weeks of age, both the wolffian and mullerian duct systems are present in the normal embryo, making male and female gonads indistinguishable. The tendency of the fetus, however, is to develop as a female, unless a Y chromosome is present. If present, a testes-determining factor induces differentiation of the gonads into testes, and female genital development is blocked. Leydig cells begin to produce testosterone, which acts on the wolffian duct to form the male internal genitalia: vas deferens, epididymis, and seminal vesicles. The testicles also produce an antimullerian hormone (AMH), also known as mullerian-inhibiting substance, which causes regression of the mullerian ducts. Formation of the phallus and scrotum (the external male genitalia) requires the conversion of testosterone to dihydrotestosterone (DHT) via 5-alpha-reductase and the presence of a specific androgen receptor. The testes migrate into the scrotum later in gestation. If the Y chromosome is absent and there are two intact and normal functioning X chromosomes, the undifferentiated gonads of an embryo will develop into female organs. The wolffian duct degenerates in the absence of androgens, and because no AMH is produced, internal female structures such as the fallopian tubes, uterus, and upper vagina develop from the müllerian duct. Similarly, in the absence of DHT, external genitalia do not fuse, thus allowing the folds and swellings to become labia and the genital tubercle to become the clitoris. The external genitalia of the female infant described in the vignette are consistent with prenatal exposure to androgens or another virilizing drug. Although the labia

http://emb.aap.org/courseprodv2/Index.asp[3/29/2012 12:00:00 PM]

Education Module Learner

majora  of most term female infants are swollen, the accompanying hyperpigmentation and rugae, as described for the infant in the vignette, are signs of masculinization. However, the normal-size clitoris and the easily visible urethra and vaginal introitus exclude the diagnosis of ambiguous genitalia. Estrogen exposure may cause breast enlargement in the term infant and withdrawal vaginal bleeding in the female infant, but it will not affect the appearance of the external genitalia. The long-acting progestin medroxyprogesterone also will not alter the appearance of the genitalia. Although an illicit substance such as marijuana and medications such as thyroid hormone may affect the fetus in general, neither will affect genital development, resulting in the physical findings described in the vignette. References: American Academy of Pediatrics. Committee on Genetics. Evaluation of the newborn with developmental anomalies of the external genitalia. Pediatrics. 2000;106:138142 Anhalt H, Neely EK, Hintz RL. Ambiguous genitalia. Pediatr Rev. 1996;17:213-220 Content Specification(s): Understand normal fetal sexual differentiation.

 

 

http://emb.aap.org/courseprodv2/Index.asp[3/29/2012 12:00:00 PM]

 

  

Education Module Learner

Questions

January 04

Asessment Summary

CME Credit Expired

Page     1  2  3  4  5  6  7  8  9  10     Help  |  Table of Contents

Overview

1

January 04

A 4,200 g newborn infant has little movement of the right arm. Perinatal history includes difficult prolonged labor, shoulder dystocia, vertex presentation, and traction on the neck during delivery. Physical examination of the right arm reveals an adducted shoulder, internally rotated upper arm, extended elbow, pronated forearm, and flexed wrist. The infant has mild respiratory distress.

2

February 04

Of the following, the MOST likely diagnosis is:

3

March 04

4

April 04

5

May 04

6

June 04

7

July 04

8

August 04

9

September 04

Assessment 04

10 October 04 November 04 December 12 04 11

NeoReviews Basic Self Assessment Return to NeoReviews.org

Erb palsy fracture of right clavicle Klumpke palsy osteomyelitis of the humerus syphilitic periosteitis

You selected

, the correct answer is

.

Erb palsy is caused by traction injury to the upper brachial plexus. The site of injury is localized to the cervical spinal nerves (C5, C6, C7). The affected limb shows little or no spontaneous movement and is held in a position of shoulder adduction, elbow extension, forearm pronation, and wrist flexion. The grasp reflex is present, but the biceps reflex is absent and the Moro reflex is limited to hand movement. There often is a sensory deficit of the affected arm. If the cervical spinal nerve (C4) also is injured, diaphragmatic paralysis from phrenic nerve injury occurs on the ipsilateral side and may cause respiratory distress. The history of difficult labor and delivery, including traction on the neck, and the physical findings of the infant described in the vignette are most consistent with the diagnosis of Erb Palsy. Klumpke palsy is caused by traction injury to the lower brachial plexus. The site of injury is localized to the cervical (C8) and thoracic spinal nerves (T1). In Klumpke palsy there is weakness of the hand, particularly the flexors of the wrist and fingers. The grasp reflex is absent. However, isolated Klumpke palsy is rare; most infants who have Klumpke palsy also have Erb palsy. This total brachial plexus palsy is associated with both proximal and distal involvement of the affected limb and more extensive sensory deficit. Horner syndrome, resulting from cervical sympathetic nerve injury, frequently accompanies lower brachial plexus injury. The syndrome consists of miosis, partial ptosis, slight enophthalmos, and anhidrosis of the ipsilateral side of the face. Brachial plexus palsy occurs in 0.5 to 2.0 per 1,000 live births. Erb palsy accounts for approximately 90% of cases, and total brachial plexus palsy accounts for most of the remainder. Diaphragmatic paralysis is associated with Erb palsy in approximately 5% of cases. Horner syndrome is associated with Klumpke palsy in approximately 30% of cases. Brachial plexus palsy is bilateral in approximately 10% of cases. Approximately 90% of infants who have brachial plexus palsy recover completely. The prognosis for recovery of function is determined by the severity and extent of nerve injury. In mild forms of nerve injury, the nerve sheath is injured, but the axones and roots remain intact, and the likelihood of spontaneous recovery is good. In moderate forms of nerve injury, the axones are severed, but the roots are intact,

http://emb.aap.org/courseprodv2/Index.asp[3/29/2012 12:00:23 PM]

Education Module Learner

and the likelihood of spontaneous recovery is fair. In severe forms of nerve injury, the roots are avulsed from the cord, and there often is associated cord injury. Spontaneous recovery in these cases is rare. Improvement in function is most rapid within the first month, but may continue for up to 1 year after birth. Fracture of the clavicle is a common traumatic musculoskeletal lesion of the newborn. The incidence is estimated at 5 per 1,000 vertex deliveries and 160 per 1,000 breech deliveries. Discoloration, tenderness, and crepitation at the fracture site are common. Although there is paucity of movement on the affected side due to irritation, there is no true motor or sensory deficit, and reflexes are normal. Osteomyelitis of the humerus is extremely rare. Signs of sepsis may be present as well as localized tenderness, erythema, and swelling of the arm. Spontaneous movement of the involved extremity is limited by pain and discomfort. Most infants who have osteomyelitis of the proximal humerus also have septic arthritis of the ipsilateral shoulder joint. There is no true motor or sensory deficit, and the reflexes are intact. Syphilitic periosteitis may present with pseudoparalysis of the limbs. Involvement of the lower extremities is much more common and usually is bilateral. Other stigmata of congenital syphilis, such as mucocutaneous lesions, hepatosplenomegaly, snuffles, and perianal condylomata, often are present. The diagnosis is confirmed by plain radiography and serologic testing for syphilis. References: Molnar GE. Brachial plexus injury in the newborn. Pediatr Rev. 1984;6:110-115 Volpe JJ. Injuries of extracranial, cranial, intracranial, spinal cord, and peripheral nervous system structures. In: Neurology of the Newborn. 3rd ed. Philadelphia, Pa: WB Saunders Co; 1995:769-792 Content Specification(s): Understand the outcome of cervical root injuries and brachial plexopathy.

 

 

http://emb.aap.org/courseprodv2/Index.asp[3/29/2012 12:00:23 PM]

 

  

Education Module Learner

Questions

February 04

Asessment Summary

CME Credit Expired

Page     1  2  3  4  5  6  7  8  9  10     Help  |  Table of Contents

Overview Assessment 04

Please remember that you must answer all 10 of the questions in order to claim CME credit for this month.

1

January 04

A 3.5 kg infant who was born at home has angulation deformities of the lower extremities. Radiographic studies reveal multiple fractures.

2

February 04

Of the following, the MOST likely cause of this infant's fractures is

3

March 04

4

April 04

5

May 04

6

June 04

hypophosphatasia

7

July 04

intrauterine compression

8

August 04

osteogenesis imperfecta

9

September 04

10 October 04 November 11 04 December 12 04

NeoReviews Basic Self Assessment Return to NeoReviews.org

achondroplasia birth trauma

You selected

, the correct answer is

.

The finding of multiple fractures in a newborn almost always is associated with osteogenesis imperfecta (OI), a class of disorders that result from defects in collagen synthesis. OI type III, which can occur in both autosomal dominant and autosomal recessive forms, presents with multiple fractures at birth in an infant who usually is born at or near term. Some affected infants will have prenatal longitudinal growth deficiency. Blue sclerae also may be evident in the neonatal period, and skull radiographs may reveal the presence of wormian bones (undermineralized calvarium), although this finding may not be evident for several weeks or months. All affected patients experience poor growth and progressive kyphoscoliosis that leads to respiratory compromise in most cases. Early mortality can be due to severe skeletal deformity, pulmonary hypertension, and cardiac failure. The underlying defect is the mutation of one of two genes, COL1A1 or COL1A2, which encode the pro-a-1(I) and pro-a-2(I) chains of type I collagen. The recessive form of OI type III is less common, except among South African blacks. In most families, recurrence has been rare, suggesting that most cases result from new dominant mutations. Prenatal identification of affected fetuses by ultrasonography is possible. Achondroplasia, the most common chondroplasia, presents in the newborn period with reduced longitudinal growth, short limbs, and macrocephaly. Fractures are not present. Birth trauma is unlikely to result in fractures of both lower extremities. Hypophosphatasia is an autosomal recessive disorder that results from the deficiency of alkaline phosphatase. It is characterized by lack of ossification and presents with bowed extremities and thoracic skeletal abnormalities that usually lead to respiratory insufficiency and death in the newborn period. Intrauterine compression can cause deformations, including clubfoot, but not skeletal fractures. References: Byers PH. Disorders of collagen biosynthesis and structure. In: Scriver CR, Beaudet AL, Sly WS, Valle D, eds. The Metabolic and Molecular Bases of Inherited Disease. 7th ed. New York, NY: McGraw Information Services Co; 1995:4029-4078 Content specification(s): Know the clinical features and know how to manage skeletal dysplasias, such as achondrogenesis, achondroplasia, chondrodermal dysplasia, epiphyseal dysostosis, osteogenesis imperfecta, hypophosphatasia, etc.

http://emb.aap.org/courseprodv2/Index.asp[3/29/2012 12:01:16 PM]

Education Module Learner

 

 

http://emb.aap.org/courseprodv2/Index.asp[3/29/2012 12:01:16 PM]

 

  

Education Module Learner

Questions

February 04

Asessment Summary

CME Credit Expired

Page     1  2  3  4  5  6  7  8  9  10     Help  |  Table of Contents

Overview Assessment 04

A 3-day-old infant presents to the emergency department with vomiting, lethargy, hypotonia, and jaundice. Physical examination reveals hepatomegaly and neurologic depression. A full sepsis evaluation is undertaken, and the Gram stain of the cerebrospinal fluid reveals gram-negative organisms.

1

January 04

Of the following, the BEST additional laboratory test to obtain is

2

February 04

3

March 04

4

April 04

5

May 04

6

June 04

7

July 04

plasma very long-chain fatty acids

8

August 04

stool porphyrins

9

September 04

10 October 04 November 04 December 12 04 11

NeoReviews Basic Self Assessment Return to NeoReviews.org

erythrocyte galactose-1-phosphate liver glycogen content plasma insulin level

You selected

, the correct answer is

.

The combination of vomiting, lethargy, neurologic depression, jaundice, hepatomegaly, and gram-negative sepsis in an infant is highly suggestive of galactosemia. This condition results from deficiency of the enzymatic activity of galactose-1-phosphate uridyltransferase and the subsequent inability to metabolize galactose. The diagnosis can be confirmed by direct measurement of the enzyme in erythrocytes and quantitation of erythrocyte galactose-1-phosphate. The clinical course of galactosemia in affected infants can be fulminant, which has led to its inclusion in the newborn screening programs of some states. Untreated patients suffer an early demise or have mental retardation if they survive. Treatment includes restriction of galactose from the diet, although affected infants typically require additional forms of therapy to correct the hyperbilirubinemia. Unless appropriate dietary therapy is initiated, the clinical response to antibiotic therapy in infants who have gram-negative sepsis may be poor. Parents of affected infants should be counseled about the recurrence risk in future pregnancies because the disorder is inherited as an autosomal recessive trait. The recurrence risk is 25%, and prenatal diagnosis is available. The glycogen storage diseases that are accompanied by hepatomegaly usually present later in infancy and are not associated with gram-negative infections. Hyperinsulinemia usually presents with hypoglycemia, which may manifest as a seizure in the neonatal period. Hepatomegaly and sepsis are not associated features. Plasma long-chain fatty acids are elevated in the peroxisomal disorders. Among these, Zellweger syndrome can present at birth with profound hypotonia, dysmorphic features, hepatomegaly, and congenital cataracts. Sepsis and presentation after the immediate newborn period are not typical. Stool porphyrin measurement is useful in the diagnosis of the porphyrias, which typically present later in childhood and are not characterized by severe illness in infancy. References: Schweitzer S, Shin Y, Jakobs C, Brodehl J. Long-term outcome in 134 patients with galactosaemia. Eur J Pediatr. 1993;152:36-43 Segal S, Berry GT. Disorders of galactose metabolism. In: Scriver CR, Beaudet AL, Sly WS, Valle D, eds. The Metabolic and Molecular Bases of Inherited Disease. 7th

http://emb.aap.org/courseprodv2/Index.asp[3/29/2012 12:01:33 PM]

Education Module Learner

ed. New York, NY: McGraw Information Services Co; 1995:967-1000 Content specifications(s): Understand the clinical manifestations, laboratory features, and treatment of disorders in the metabolism of carbohydrates (excluding glucose)

 

 

http://emb.aap.org/courseprodv2/Index.asp[3/29/2012 12:01:33 PM]

 

  

Education Module Learner

Questions

February 04

Asessment Summary

CME Credit Expired

Page     1  2  3  4  5  6  7  8  9  10     Help  |  Table of Contents

Overview Assessment 04

A pediatrician calls you regarding a 1-month-old boy who has had inspiratory stridor since birth. The stridor is associated with retractions when the infant becomes agitated. Physical examination reveals a weight of 3.4 kg (10th percentile), no expiratory stridor, and weak cry.

1

January 04

Of the following, the MOST likely cause of the stridor in this infant is

2

February 04

3

March 04

4

April 04

5

May 04

6

June 04

7

July 04

subglottic cyst

8

August 04

tracheomalacia

9

September 04

10 October 04 November 04 December 12 04 11

NeoReviews Basic Self Assessment Return to NeoReviews.org

bilateral paralysis of the vocal cords laryngeal cleft laryngomalacia

You selected

, the correct answer is

.

Stridor is produced by a rapid, turbulent flow of air through a narrowed segment of the airway. It is an important sign of airway obstruction in infants and children. Stridor may be classified as inspiratory, expiratory, or biphasic. Inspiratory stridor typically is produced by an obstruction at or above the level of the vocal cords. Biphasic stridor most commonly occurs with obstruction at the glottic or subglottic level. The subglottic lumen is surrounded by the cricoid cartilage, which is the only fixed ring of cartilage in the airway. Inspiration and expiration produce similar turbulent flow when this area is obstructed, resulting in biphasic stridor. Expiratory stridor most commonly is produced by an intrathoracic site of airway obstruction. In addition to stridor, other physical findings consistent with airway obstruction include retractions, especially with glottic and supraglottic disease, hoarseness, tachypnea, nasal flaring, difficulty feeding, intermittent cyanosis, and failure to thrive. The differential diagnosis of congenital stridor includes laryngomalacia, tracheomalacia, bilateral vocal cord paralysis, subglottic stenosis, and a variety of obstructing lesions, such as tumors or cysts. Laryngomalacia is by far the most common cause of congenital stridor, occurring in approximately 70% to 80% of patients who have stridor. It is characterized by an intermittent whooping, late inspiratory stridor that typically involves no respiratory distress. Vocal cord paralysis is the second most common cause of stridor. Unilateral paralysis may cause a hoarse, breathy voice, but it rarely causes airway distress. Bilateral vocal cord paralysis presents with inspiratory stridor that often is severe and is associated with retractions and blue spells. The cry may be normal or weak, as described for the infant in the vignette. Subglottic stenosis manifests as biphasic stridor with no voice abnormalities and may be congenital or acquired following intubation. Subglottic cysts also can occur after intubation. They rarely are present at birth, and commonly they cause progressive stridor 1 to 2 months after intubation. The stridor is typically biphasic and may be associated with voice abnormalities. Laryngeal cleft is a rare congenital defect of the posterior larynx in which the dorsal aspect of the cricoid cartilage or trachea fails to fuse. Laryngeal clefts can be

http://emb.aap.org/courseprodv2/Index.asp[3/29/2012 12:01:51 PM]

Education Module Learner

associated with stridor if there is concomitant subglottic stenosis or redundancy of supraglottic tissues. Most commonly, laryngeal clefts cause aspiration, recurrent respiratory infections, difficulty swallowing, and choking spells. Tracheomalacia, or flaccidity and collapse of tracheal cartilage, classically is associated with expiratory stridor. Usually there are no abnormalities of the voice. In severe cases, failure to thrive and cyanosis may be present. The severe congenital inspiratory stridor that is associated with retractions and a weak cry, as described for the infant in the vignette, suggests the diagnosis of vocal cord paralysis. An accurate diagnosis usually can be made by flexible upper airway endoscopy with visualization of the larynx and trachea. Flexible endoscopy can be used to assess the supraglottic larynx and to diagnose both laryngomalacia and vocal cord paralysis. The subglottic lumen, however, may be difficult to assess with this technique. Tracheomalacia is readily apparent on either flexible or rigid bronchoscopy. Airway fluoroscopy can be useful in diagnosing vocal cord paralysis and tracheomalacia. Supraglottic abnormalities such as laryngomalacia are difficult to determine on radiography. Microlaryngoscopy and rigid bronchoscopy in the operating room often are required to make a diagnosis of laryngeal cleft or significant subglottic stenosis. A barium swallow may demonstrate aspiration in a cleft larynx. Vascular rings (eg, double aortic arch [Figure 26A], retroesophageal subclavian) cause esophageal compression that may be noted on barium swallow. Magnetic resonance imaging or computed tomography of the chest also can be performed to evaluate the thoracic vascular anatomy. References: Brodsky L. Congenital stridor. Pediatr Rev. 1996;17:408-411 Holinger LD. Etiology of stridor in the neonate, infant and child. Ann Otol Rhinol Laryngol. 1980;89:397-400 Stern RC. Congenital anomalies. In: Behrman RE, Kliegman RM, Nelson WE, eds. Nelson Textbook of Pediatrics. 16th ed. Philadelphia, Pa: WB Saunders Co; 2000:1271-1274 Content specification(s) Know the various causes of stridor in the newborn

 

 

http://emb.aap.org/courseprodv2/Index.asp[3/29/2012 12:01:51 PM]

 

  

Education Module Learner

Questions

February 04

Asessment Summary

CME Credit Expired

Page     1  2  3  4  5  6  7  8  9  10     Help  |  Table of Contents

Overview Assessment 04

During prenatal care screening tests yield a serum alpha-fetoprotein value is markedly increased. Of the following, the MOST appropriate advice to give to this woman is that she should:

1

January 04

2

February 04

have an ultrasonographic examination to date her pregnancy and to search for fetal anomalies

3

March 04

have another blood sample drawn to repeat the test

4

April 04

have chorionic villus sampling as soon as possible to determine the chromosome complement of the fetus

5

May 04

6

June 04

7

July 04

8

August 04

9

September 04

10 October 04 November 04 December 12 04 11

NeoReviews Basic Self Assessment Return to NeoReviews.org

obtain further testing only if she older than age 35 only be concerned if there is a history of open neural tube defects in her family

You selected

, the correct answer is

.

Alpha-fetoprotein (AFP) is produced by the fetal liver and crosses the placenta to enter the maternal circulation. Any defect that causes a breech in fetal skin can result in increased levels of AFP in the maternal circulation due to leaking of the protein. These fetal defects include open neural tube defects, anencephaly, and omphalocele. Maternal serum AFP (MSAFP) levels also can be increased in twin pregnancies, with fetal demise, and in pregnancies in which the fetus has congenital nephrosis. Because the measurement of MSAFP is simple and inexpensive, the American College of Obstetricians and Gynecologists, the American Society of Human Genetics, and the American Academy of Pediatrics recommend offering MSAFP screening to all pregnant women at 16 to 18 weeks of gestation. However, such screening should be undertaken only if there is adequate counseling, access to high-quality laboratory services, and appropriate facilities for follow-up testing (ie, qualified diagnostic centers that offer conventional and high-resolution ultrasonography and amniocentesis). More recently, additional biochemical markers have been added to this screening test to permit the identification of pregnancies at increased risk for chromosomal abnormalities, most notably trisomy 21. Depending on the cutoff used by the laboratory to define an elevated level, which is usually 2 to 2.5 times the median value for gestational age, MSAFP screening detects most fetuses that have open neural tube defects. However, because it is a screening test, MSAFP results are abnormal in approximately 1% to 5% of pregnant women. Because the incidence of open neural tube defects is generally 1 in 1,000 or less, most findings of elevated MSAFP are due to other reasons (eg, incorrect dating of the pregnancy, other congenital anomalies, intrauterine growth retardation, multiple gestations, fetal demise). Incorrect dating is the most common reason for a falsely positive MSAFP. If the dating of the pregnancy is changed by ultrasonography, the MSAFP value should be recalculated to reassess the risk. If the dating is correct, the patient should be offered high-resolution fetal ultrasonography to search for anomalies. If ultrasonography does not provide an explanation for the abnormal result (as occurs in about 50% of cases), amniocentesis should be offered to measure amniotic fluid AFP (AFAFP). Some laboratories request a second sample after an initial elevated MSAFP level; others proceed immediately to follow-up ultrasonography. In general, second samples should be obtained only if the initial MSAFP concentration is minimally elevated and there is sufficient time for processing a second specimen.

http://emb.aap.org/courseprodv2/Index.asp[3/29/2012 12:02:51 PM]

Education Module Learner

Clinical trials in pregnant women who have had a prior pregnancy affected by a neural tube defect have demonstrated that folic acid supplements substantially reduce the risk of recurrent neural tube defects. In one such trial, administration of 4 mg of folic acid daily beginning at least 1 month before conception through the first trimester reduced the recurrence risk of neural tube defects from 3.5% to 1.0%. Trials in women who have not had a prior affected pregnancy also have shown a beneficial effect. It is now recommended that all women of childbearing age take folic acid supplements to prevent the occurrence of neural tube defects. All women still should undergo MSAFP screening because there is no evidence to suggest that folic acid supplements can prevent all neural tube defects. Because elevated MSAFP levels are associated primarily with open neural tube defects, which usually do not result from chromosome abnormalities, chorionic villus sampling, which is used to determine fetal karyotype, would not be indicated for the woman in the vignette. In addition, AFP cannot be measured in chorionic villi. Open neural tube defects are among the most common birth defects, occurring in approximately 1 in 1,000 pregnancies. In most cases, there is no family history, although those who do have a positive family history may be at increased risk over the general population. There is no association of neural tube defects with maternal age. References: American Academy of Pediatrics Committee on Genetics. Maternal serum alphafetoprotein screening. Pediatrics. 1991;88:1282-1283 American College of Obstetricians and Gynecologists. Alpha-fetoprotein. In: ACOG Technical Bulletin: An Educational Aid to Obstetricians-Gynecologists. Washington, DC: American College of Obstetricians and Gynecologists; 1991:No. 154 Recommendations for the use of folic acid to reduce the number of cases of spina bifida and other neural tube defects. MMWR Morb Mortal Wkly Rep. 1992;41(RR-14):17 Werler MM, Shapiro S, Mitchel AA. Periconceptional folic acid exposure and risk of occurrent neural tube defects. JAMA. 1993;269:1257-1261 Content specification(s): Understand the significance of abnormal maternal values on a second trimester multiscreen or alpha-fetoprotein test

 

 

http://emb.aap.org/courseprodv2/Index.asp[3/29/2012 12:02:51 PM]

 

  

Education Module Learner

Questions

February 04

Asessment Summary

CME Credit Expired

Page     1  2  3  4  5  6  7  8  9  10     Help  |  Table of Contents

Overview Assessment 04

You are discussing with a medical student the factors affecting the energy requirements of a preterm neonate. Of the following, the MOST accurate statement about energy expenditure of a preterm infant is that:

1

January 04

2

February 04

a thermoneutral environment is effective in minimizing energy expenditure

3

March 04

diet-induced thermogenesis is higher with continuous than with intermittent feeding

4

April 04

5

May 04

6

June 04

physical activity is the major component of total energy expenditure

7

July 04

resting metabolic rate decreases with advancing postnatal age

8

August 04

9

September 04

10 October 04 November 04 December 12 04 11

NeoReviews Basic Self Assessment Return to NeoReviews.org

metabolic cost of growth is higher with deposition of fat than with synthesis of protein

You selected

, the correct answer is

.

The preterm neonate's energy expenditure can be minimized by keeping the infant in a thermoneutral environment. The thermoneutral environment is a range of ambient temperatures within which the metabolic rate of the infant is minimal and the infant can maintain a normal body temperature without any regulatory changes in metabolic heat production or evaporative heat loss. The thermoneutral range varies with gestational age. In an unclothed resting adult, the lower limit of the thermoneutral range is 26C to 28C (78.8F to 82.4F) in an environment of 50% relative humidity and still air. Under similar conditions, the lower limit of the thermoneutral range is 32C (89.6° F) or higher in a naked term neonate and 35C (95F) or higher in a naked preterm neonate. Diet-induced thermogenesis, also known as specific dynamic action, thermic effect of food, or postprandial thermogenesis, is the increase in metabolic rate that follows food intake. It represents the energy consumption necessary for digestion, absorption, and assimilation of nutrients. The magnitude of increase in energy expenditure following the ingestion of nutrients is estimated to vary between 4% and 30% in both term and preterm neonates. Diet-induced thermogenesis is lower with continuous than with intermittent enteral feeding. Metabolic cost of growth represents the energy required for the formation of new tissue, and it varies with the composition of the synthesized tissue. The cost of depositing absorbed dietary fat into adipose tissue is much less than that of synthesizing new protein. The overall metabolic cost of growth in neonates is estimated at approximately 4.4 kcal/g of weight gain. Energy expenditure increases with physical activity, but because neonates sleep 80% to 90% of the time, physical activity is a small component of their energy expenditure compared with that of adults. It is estimated that physical activity contributes to only about 10% of the total energy expenditure in preterm neonates. Resting metabolic rate increases steadily from birth in both term and preterm neonates. The resting metabolic rate in term neonates is estimated at approximately 40 kcal/kg per day at 3 days of postnatal age, increasing to approximately 60 kcal/kg per day at 3 months of postnatal age. A similar but smaller increase in resting metabolic rate is observed in preterm neonates.

http://emb.aap.org/courseprodv2/Index.asp[3/29/2012 12:14:53 PM]

Education Module Learner

References: Brück K. Neonatal thermal regulation. In: Polin RA, Fox WW, eds. Fetal and Neonatal Physiology. 2nd ed. Philadelphia, Pa: WB Saunders Co; 1998:676-702 Leitch CA, Denne SC. Energy expenditure in the extremely low-birth weight infant. Clin Perinatol. 2000;27:181-195   Content Specification(s):  Understand the coloric cost of physical activity. Understand the caloric cost of maintaining body temperature.

 

 

http://emb.aap.org/courseprodv2/Index.asp[3/29/2012 12:14:53 PM]

 

  

Education Module Learner

Questions

February 04

Asessment Summary

CME Credit Expired

Page     1  2  3  4  5  6  7  8  9  10     Help  |  Table of Contents

Overview Assessment 04

A newborn female has loose neck skin and nonpitting edema of the lower extremities. Of the following, the MOST appropriate evaluation for this infant is:

1

January 04

2

February 04

3

March 04

magnetic resonance imaging of the brain

4

April 04

slitlamp ophthalmologic examination

5

May 04

6

June 04

7

July 04

8

August 04

9

September 04

10 October 04 November 04 December 12 04 11

NeoReviews Basic Self Assessment Return to NeoReviews.org

blood chromosome analysis

ultrasonography of the liver voiding cystourethrography

You selected

, the correct answer is

.

The finding of loose neck skin, which is suggestive of the presence of a cystic hygroma in fetal life, and nonpitting edema of the lower extremities in a newborn female should raise the suspicion of Turner syndrome. Congenital lymphedema, which occurs in up to 80% of affected females, typically disappears during infancy, leaving only a puffy appearance to the hands and feet, although in some patients it reappears when estrogen therapy is initiated. The posterior neck skin can persist as the pterygium colli, or webbed neck. The clinical diagnosis of Turner syndrome should be confirmed by peripheral blood chromosome analysis, which will reveal monosomy X, mosaic monosomy X, or the presence of an abnormal X chromosome that contains a deletion. Affected girls also have short stature, ovarian dysgenesis, broad chest with widespaced nipples, ear anomalies, cubitus valgus, and renal and cardiac defects. Intelligence is normal. Estrogen replacement therapy at the expected time of puberty is required in most cases, and treatment with growth hormone also should be offered. Magnetic resonance imaging of the brain, slitlamp ophthalmologic examination, and ultrasonography of the liver are not indicated because central nervous system and ophthalmologic findings are not features of Turner syndrome, and liver size and function is normal. Although renal defects may be present, the most common abnormality is horseshoe kidney, which can be detected by ultrasonography and usually has no clinical effects that would prompt the need for voiding cystourethrography. References: Robinson A, Bender BG, Linden MG, Salbenblatt JA. Sex chromosome aneuploidy: the Denver Prospective Study. Birth Defects Orig Artic Ser. 1990;26:59-115 Rosenfeld RG, Tesch LG, Rodriguez-Rigua LJ, et al. Recommendations for diagnosis, treatment, and management of individuals with Turner syndrome. Endocrinologist. 1994;4:351-358 Content Specification(s): Recognize the physical characteristics and chromosomal pattern of sex chromosome aneuploidy.

http://emb.aap.org/courseprodv2/Index.asp[3/29/2012 12:15:12 PM]

Education Module Learner

 

 

http://emb.aap.org/courseprodv2/Index.asp[3/29/2012 12:15:12 PM]

 

  

Education Module Learner

Questions

February 04

Asessment Summary

CME Credit Expired

Page     1  2  3  4  5  6  7  8  9  10     Help  |  Table of Contents

Overview

1

January 04

You are reviewing results of head ultrasonography performed on a 6-week-old extremely low-birthweight infant. The scan shows a large, encapsulated, fluid-filled cavity in the right cerebrum. The infant has required prolonged ventilator assistance and parenteral nutrition through a central venous catheter. She also has received broad-spectrum antibiotic therapy for multiple episodes of suspected sepsis.

2

February 04

Of the following, the MOST likely organism to cause this brain lesion is:

3

March 04

4

April 04

5

May 04

6

June 04

7

July 04

8

August 04

9

September 04

Assessment 04

10 October 04 November 04 December 12 04 11

NeoReviews Basic Self Assessment Return to NeoReviews.org

Citrobacter diversus Clostridium perfringens Gardnerella vaginalis Leptospira canicola Yersinia enterocolitica

You selected

, the correct answer is

.

Citrobacter diversus causes sporadic and epidemic neonatal sepsis and meningitis. It is uniquely associated with brain abscesses, which usually are multiloculated and located in the cerebrum. The pathogenesis of brain abscess caused by C diversus involves cerebral vasculitis, with infarction and bacterial invasion of the necrotic tissue. Early manifestations of brain abscess include increased intracranial pressure, focal cerebral signs, and meningitis. The disease is associated with a high fatality rate, and mental retardation is common among survivors. Clostridium perfringens rarely is isolated in the neonate. Infection with this pathogen usually presents as omphalitis, localized cellulitis, necrotizing fasciitis, or generalized septicemia. Clostridial sepsis may include active hemolysis, hyperbilirubinemia, and hemoglobinuria. Gardnerella vaginalis is a gram-negative coccobacillus that is present in the genital tracts of up to one third of pregnant women. Infection in the neonate usually occurs in early postnatal life. Most infants are asymptomatic, but some may have signs and symptoms of congenital pneumonia. Leptospira canicola infection also is rare in the neonate. The organism usually is acquired by the transplacental route and largely affects the liver in the neonate. Yersinia enterocolitica is a rare pathogen in the neonate. Infection with this pathogen causes watery diarrhea, fecal blood, and other symptoms and signs of gastroenteritis. Rarely, autoimmune manifestations, such as arthritis and erythema nodosum, may occur. References: Klein JO, Marcy SM. Bacterial sepsis and meningitis. In: Remington JS, Klein JO, eds. Infectious Diseases of the Fetus and Newborn Infant. 4th ed. Philadelphia, Pa: WB Saunders Co; 1995:835-890 Marcy SM, Overturf GD. Focal bacterial infections. In: Remington JS, Klein JO, eds. Infectious Diseases of the Fetus and Newborn Infant. 4th ed. Philadelphia, Pa: WB Saunders Co; 1995:935-979

http://emb.aap.org/courseprodv2/Index.asp[3/29/2012 12:15:48 PM]

Education Module Learner

Pickering LK, Guerrant RL, Cleary TG. Microorganisms responsible for neonatal diarrhea. In: Remington JS, Klein JO, eds. Infectious Diseases of the Fetus and Newborn Infant. 4th ed. Philadelphia, Pa: WB Saunders Co; 1995:1142-1222   Content Specification(s): Understand the causes, clinical manifestations, laboratory features, treatment, and complications of meningitis and menigoencephalitis.

 

 

http://emb.aap.org/courseprodv2/Index.asp[3/29/2012 12:15:48 PM]

 

  

Education Module Learner

Questions

February 04

Asessment Summary

CME Credit Expired

Page     1  2  3  4  5  6  7  8  9  10     Help  |  Table of Contents

Overview Assessment 04

A newborn infant is delivered by emergent cesarean section at 41 weeks' gestation following a pregnancy complicated by a prolapsed umbilical cord and meconiumstained amniotic fluid. At 6 hours of age, the infant has a generalized tonic-clonic seizure.

1

January 04

Of the following, the MOST likely explanation for this seizure is:

2

February 04

3

March 04

4

April 04

5

May 04

6

June 04

7

July 04

meningitis

8

August 04

pyridoxine dependency

9

September 04

10 October 04 November 04 December 12 04 11

NeoReviews Basic Self Assessment Return to NeoReviews.org

hyponatremia hypoxic-ischemic encephalopathy intracranial hemorrhage

You selected

, the correct answer is

.

The history of umbilical cord prolapse and the meconium staining of amniotic fluid described for the infant in the vignette suggest perinatal asphyxia. Hypoxicischemic encephalopathy (HIE) due to perinatal asphyxia is the single most common cause of neonatal seizures in the term infant. HIE accounts for approximately 60% of neonatal seizures. The first seizure occurs within 24 hours after birth in most infants who have HIE and typically is clonic and multifocal. The clonic movements are rhythmic and slow (approximately one to three jerks per second) and migrate in a variable fashion to involve several body parts. Most infants also exhibit subtle seizures that are characterized by tonic horizontal deviation of eyes or sustained eye opening with ocular fixation, oral-buccal-lingual movements such as sucking and smacking, rowing movements of limbs, and apneic spells. Pupillary size varies, although dilated reactive pupils tend to predominate in infants who have mild HIE and constricted reactive pupils are common in those who have moderate-to-severe HIE. Severely affected infants often are stuporous or comatose, show diffuse hypotonia, and have status epilepticus. Electroencephalographic (EEG) abnormalities are common in infants who have moderate-to-severe HIE. Among the metabolic causes of neonatal seizure, abnormalities of glucose, calcium, and magnesium metabolism are the most common, but they account for no more than 6% of neonatal seizures. Hyponatremia (serum sodium concentration of 40°C [>104°F]) has been reported to be associated with a variety of structural abnormalities in the fetus, including neural tube defects, microcephaly, and clefting, but not with syndactyly or amputations. Trisomy 13 is characterized by

http://emb.aap.org/courseprodv2/Index.asp[3/29/2012 2:45:22 PM]

Education Module Learner

severe central nervous system abnormalities, clefting, cardiac defects, and polydactyly. References: Jones KL. Smith's Recognizable Patterns of Human Malformation. 5th ed. Philadelphia, Pa: WB Saunders Co; 1997 Moerman P, Fryns JP, Vandenberghe K, Lauweryns JM. Constrictive amniotic bands, amniotic adhesions, and limb-body wall complex: discrete disruption sequences with pathogenetic overlap. Am J Med Genet. 1992;42:470-479 Content Specification(s): Recognize the consequences of the amniotic band syndrome Know the effect of maternal nervous system diseases, including seizures, and their treatment on the fetus

 

 

http://emb.aap.org/courseprodv2/Index.asp[3/29/2012 2:45:22 PM]

 

  

Education Module Learner

Questions

June 04

Assessment Summary

CME Credit Expired

Page     1  2  3  4  5  6  7  8  9  10     Help  |  Table of Contents

Overview Assessment 04

You are discussing with a medical student the benefits of breastfeeding in terms of preventing infectious diseases during infancy. Of the following pathogens, it is MOST likely that breastfeeding provides protection against infection caused by

1

January 04

2

February 04

cytomegalovirus

3

March 04

Escherichia coli

4

April 04

herpes simplex virus

5

May 04

6

June 04

7

July 04

8

August 04

9

September 04

10 October 04 November 04 December 12 04 11

NeoReviews Basic Self Assessment Return to NeoReviews.org

human immunodeficiency virus Mycobacterium tuberculosis

You selected

, the correct answer is

.

Human milk is protective against enteropathogenic Escherichia coli and other gastrointestinal pathogens. This protection is greatest during an infant's first 3 months of life and declines with increasing age. During the weaning period, partial breastfeeding continues to confer some protection. The mechanisms of protection are at least threefold. Breastfeeding confers protection through active components of human milk, which include cells, antibodies, carrier proteins, enzymes, and hormones. The process of breastfeeding itself may decrease exposure to enteropathogenic E coli that could be present in contaminated bottles, milk, or water. Finally, administration of human milk initiates and maintains the growth of Lactobacillus bifidus in the gut; this decreases luminal pH, which inhibits the growth of E coli. For these and other reasons, breastfeeding should be encouraged. Breastfeeding is contraindicated in women who have human immunodeficiency virus (HIV) infection because human milk is a well-known source of transmission of this virus. HIV virions have been observed by electron microscopy within histiocytes and in the cell-free fraction of human milk. Colostrum contains a higher concentration of lymphocytes and macrophages than mature milk and, therefore, is more likely to harbor cell-associated viruses such as HIV. The risk of transmission of HIV through breastfeeding is increased when infection is recent or illness is more advanced in the mother. Both of these clinical situations are associated with a high viral load in body fluids, including human milk. Human milk does not confer protection against herpes simplex virus (HSV). Perinatal HSV infection can occur by transplacental, intrapartum, or postnatal transmission. The latter is most likely to occur following contact with orolabial herpes. Postnatal transmission of HSV also can occur if breast lesions are present. Human milk does not confer protection against Mycobacterium tuberculosis. Neonatal tuberculosis is acquired in four ways: by inhalation of infected droplets (via the lung), by ingestion of infected droplets (via the gut), by contamination of traumatized skin or mucous membranes, or by ingestion of infected milk. Among these routes of acquisition, the lung is by far the most common portal of entry in the neonate. Human milk also is a source of transmission of cytomegalovirus (CMV). CMV infection is an endemic, often subclinical infection among women of low

http://emb.aap.org/courseprodv2/Index.asp[3/29/2012 2:45:44 PM]

Education Module Learner

socioeconomic status. Prolonged viral shedding of CMV is common, primarily through the genital tract and milk. The fetus can be infected by the transplacental route. However, the newborn is infected more frequently by contamination from the genital tract during birth or by ingestion of infected milk. References Kohl S. Neonatal herpes simplex virus infection. Clin Perinatol. 1997;24:129-150 Nelson CT, Demmler GJ. Cytomegalovirus infection in the pregnant mother, fetus, and newborn infant. Clin Perinatol. 1997;24:151-160 Ogra PL, Rassin DK. Human breast milk. In: Remington JS, Klein JO, eds. Infectious Diseases of the Fetus & Newborn Infant. 4th ed. Philadelphia, Pa: WB Saunders Co; 1995:108-139 Ruff AJ. Breastmilk, breastfeeding, and transmission of viruses to the neonate. Semin Perinatol. 1994;18:510-516 Smith MHD, Teele DW. Tuberculosis. In: Remington JS, Klein JO, eds. Infectious Diseases of the Fetus & Newborn Infant. 4th ed. Philadelphia, Pa: WB Saunders Co; 1995:1074-1086 Content Specification(s) Understand the immunologic constituents in human milk and their physiologic effects  

 

 

http://emb.aap.org/courseprodv2/Index.asp[3/29/2012 2:45:44 PM]

 

  

Education Module Learner

Questions

June 04

Assessment Summary

CME Credit Expired

Page     1  2  3  4  5  6  7  8  9  10     Help  |  Table of Contents

Overview Assessment 04

A newborn has respiratory distress, with difficulty breathing at rest and an inability to feed. He has a normal, vigorous cry, which relieves the distress, and no stridor. Physical examination reveals no obvious anomalies. Of the following, the MOST appropriate procedure to lead to the presumptive diagnosis is:

1

January 04

2

February 04

3

March 04

4

April 04

an attempt to pass a catheter through the nose into the nasopharynx

5

May 04

anteroposterior neck radiographs to assess subglottic lumen diameter

6

June 04

barium esophagraphy to exclude vascular ring

7

July 04

8

August 04

9

September 04

10 October 04 November 04 December 12 04 11

NeoReviews Basic Self Assessment Return to NeoReviews.org

airway fluoroscopy to evaluate vocal cord mobility

examination of the oral cavity for retrognathia and glossoptosis 

You selected

, the correct answer is

.

Respiratory distress in the newborn may be caused by obstruction at any point in the airway. Newborns who have nasal obstruction and a normal distal airway will have respiratory distress when the mouth is closed and feeding difficulties. A normal, vigorous cry and adequate airway with the mouth open, as described for the infant in the vignette, suggests a normal distal airway. Obstruction at the level of the oropharynx and hypopharynx typically manifests as inspiratory stridor. The stridor persists when the mouth is closed. Obstruction at the level of the glottis or subglottis classically causes biphasic stridor, often with retractions and an abnormal, weak, or hoarse cry if the vocal cords are involved. Tracheal obstruction classically manifests as expiratory stridor. The most common cause of congenital nasal obstruction is choanal atresia, which occurs in 1 in 7,000 live births. It is slightly more common among females and usually is unilateral. Most cases are mixed bony-membranous atresias. Approximately 50% of patients have associated anomalies, one of the most common of which is the CHARGE association that involves anomalies of choanal atresia, colobomas of the retina, cardiac defects, genitourinary anomalies, mental retardation, and hearing loss. Bilateral choanal atresia with complete nasal obstruction results in immediate onset of respiratory distress in newborns. Unilateral choanal atresia rarely causes acute respiratory distress, but often presents later in life with unilateral thick rhinorrhea that does not improve with medications. The initial diagnostic procedure of choice to evaluate for choanal atresia is to pass a #6 French catheter or dilator through the nose into the oropharynx. Failure of the catheter to pass through the nose suggests the diagnosis, which can be confirmed by flexible endoscopy. Computed tomography (CT) in the axial plane is recommended prior to surgical intervention. Most infants who have choanal atresia require immediate intervention with an oral airway. Once they are stabilized and the diagnosis is confirmed with CT scan, operative repair is performed. Most commonly, endoscopic procedures are used to open the atresia plate. Stenting of the nasal airway sometimes is needed postoperatively.

http://emb.aap.org/courseprodv2/Index.asp[3/29/2012 2:46:02 PM]

Education Module Learner

Retrognathia and glossoptosis associated with cleft of the secondary palate is a description of Pierre Robin sequence, previously called the Pierre Robin syndrome. Affected infants have upper airway obstruction and feeding difficulties because of the glossoptosis. Airway obstruction may be present, even with crying. Vascular rings may cause tracheal and esophageal compression. Expiratory stridor with respiratory distress that is unrelieved with crying and dysphagia are typical manifestations. Subglottic stenosis presents with biphasic stridor and respiratory distress. Stridor is more pronounced with agitation and crying. Bilateral vocal cord paralysis also presents with predominantly inspiratory, but occasionally biphasic stridor. Because the vocal cords typically are paralyzed in the paramedian position, the cry may be normal, but crying and agitation increase stridor and respiratory distress. Several other conditions may produce symptoms similar to those reported for choanal atresia. Anterior pyriform aperture stenosis, with narrowing of the front of nose, is seen occasionally. The formation of cysts of the distal aspect of the nasal lacrimal duct (nasal lacrimal duct cyst) can obstruct the nose and cause respiratory distress. Rhinitis of infancy, an idiopathic inflammation of the nose, can present with nasal obstruction that mimics choanal atresia. In all of these conditions, catheters typically can be passed into the oropharynx. References: Brown OE, Pownell P, Manning SC. Choanal atresia: a new anatomic classification and clinical management applications. Laryngoscope. 1996:106:97-101 Derkay CS, Grundfast KM. Airway compromise from nasal obstruction in neonates and infants. Int J Pediatr Otorhinolaryngol. 1990;19:241-249 Hengerer AS, Yanofsky SD. Congenital malformations of the nose and paranasal sinuses. In: Bluestone CD, Stool SE, Kenna MA, eds. Pediatric Otolaryngology. 3rd ed. Philadelphia, Pa: WB Saunders Co; 1996:831-842 Kenna MA. Congenital disorders of the nose. In: Behrman RE, Kliegman RM, Nelson WE, eds. Nelson Textbook of Pediatrics. 16th ed. Philadelphia, Pa: WB Saunders Co; 2000:1258-1259 Content Specification(s): Recognize the incidence, clinical manifestations, and treatment of bilateral and unilateral choanal atresia  

 

 

http://emb.aap.org/courseprodv2/Index.asp[3/29/2012 2:46:02 PM]

 

  

Education Module Learner

Questions

June 04

Assessment Summary

CME Credit Expired

Page     1  2  3  4  5  6  7  8  9  10     Help  |  Table of Contents

Overview Assessment 04

You note microcephaly, hepatosplenomegaly, and a purpuric skin rash on physical examination of a newborn. A complete blood count reveals a platelet count of 90 x 10 3 /mcL (90 x 10 9 /L) and hemoglobin of 13.1 g/dL (131 g/L).

Of the following, the MOST likely pathogen infecting this infant is

1

January 04

2

February 04

cytomegalovirus

3

March 04

herpes simplex virus

4

April 04

5

May 04

6

June 04

7

July 04

8

August 04

9

September 04

10 October 04 November 04 December 12 04 11

NeoReviews Basic Self Assessment Return to NeoReviews.org

human immunodeficiency virus rubella virus Treponema pallidum

You selected

, the correct answer is

.

The differential diagnosis of neonatal thrombocytopenia includes neonatal autoimmune thrombocytopenia, infection-induced thrombocytopenia, maternal idiopathic thrombocytopenic purpura, maternal drug exposure, and other rare causes. Although the most common cause of neonatal thrombocytopenia is alloimmune thrombocytopenia, the additional findings of microcephaly and hepatosplenomegaly suggest that the infant described in the vignette more likely has a congenital infection. The most probable pathogen among those listed is cytomegalovirus (CMV). Congenital CMV infection has a spectrum of manifestations, but it is usually asymptomatic. Approximately 5% of affected infants have profound involvement, with intrauterine growth retardation, neonatal jaundice, purpura, hepatosplenomegaly, microcephaly, brain damage, intracerebral calcifications, and retinitis. Both herpes simplex and human immunodeficiency (HIV) infections are unlikely in the newborn in the vignette because both are acquired most commonly in the perinatal period from exposure to maternal virus in secretions or blood. Neonatal herpes simplex virus infection rarely is acquired during gestation. In newborns, initial symptoms of herpes simplex virus infection can occur anytime between birth and approximately 4 weeks of age. Disseminated herpes simplex disease that involves multiple organs, most prominently the liver and lungs, has the earliest age of onset, often during the first week of life. The infant suffering from disseminated or central nervous system herpes simplex infection is unlikely to demonstrate any of the findings indicative of in utero infection, such as poor growth, microcephaly, chorioretinitis, or heart disease. Most HIV-exposed infants are born at term, have normal birthweights, and have normal physical examination findings at birth. There may be a slight increase in the incidence of preterm and low birthweight infants, although it is unclear whether HIV infection or other confounding variables account for this. Because a substantial portion of HIV infection in infants is believed to occur at delivery, physical findings such as adenopathy or hepatosplenomegaly, which later may suggest HIV infection, are usually absent in the newborn period. Infants who are small for gestational age or have microcephaly, adenopathy, or hepatosplenomegaly should receive a thorough assessment for other causative factors.

http://emb.aap.org/courseprodv2/Index.asp[3/29/2012 2:47:53 PM]

Education Module Learner

The most commonly described anomalies associated with the congenital rubella syndrome are ophthalmologic (cataracts, retinopathy, congenital glaucoma), cardiac (patent ductus arteriosus, peripheral pulmonary artery stenosis), auditory (sensorineural deafness), and neurologic (behavioral disorders, meningoencephalitis, mental retardation). In addition, affected infants frequently experience growth retardation and may have radiolucent bone disease, hepatosplenomegaly, thrombocytopenia, and purpuric skin lesions. Mild forms of the disease can be associated with few or no obvious clinical manifestations at birth. Congenital infection with Treponema pallidum (syphilis) can result in stillbirth, hydrops fetalis, or prematurity; at birth, infants may have signs of disease or they may present at up to 2 years of age with hepatosplenomegaly, snuffles, lymphadenopathy, mucocutaneous lesions, osteochondritis and pseudoparalysis, edema, rash, hemolytic anemia, and thrombocytopenia. Some infected infants who are asymptomatic, if untreated or inadequately treated, may develop these signs within the first weeks of life.

References: Baley JE, Toltzis P. Viral infections. In: Fanaroff AA, Martin RJ, eds. Neonatal-Perinatal Medicine: Diseases of the Fetus and Infant. 7th ed. St. Louis, Mo: Mosby, Inc; 2002:755802 Cole FS, Taeusch HW. Infections with spirochetal and parasitic organisms. In: Taeusch HW, Ballard RA, Avery ME, eds. Avery's Diseases of the Newborn. 7th ed. Philadelphia, Pa: WB Saunders Co; 1998:535-540 Cole FS. Viral infections of the fetus and newborn. In: Taeusch HW, Ballard RA, Avery ME, eds. Avery's Diseases of the Newborn. 7th ed. Philadelphia, Pa: WB Saunders Co; 1998:467-489 Freij BJ, Sever JL. Chronic infections. In: Avery GB, Fletcher MA, MacDonald MG, eds. Neonatology: Pathophysiology and Management of the Newborn. 5th ed. Philadelphia, Pa: JB Lippincott Co; 1999:1123-1188 Luchtman-Jones L, Schwartz AL, Wilson DB. Hematologic problems in the fetus and neonate. In: Fanaroff AA, Martin RJ, eds. Neonatal-Perinatal Medicine: Diseases of the Fetus and Infant. 7th ed. St. Louis, Mo: Mosby, Inc; 2002:1183-1238 Content Specification(s): Understand the epidemiology, pathogenesis, prevention, clinical manifestations, diagnostic criteria, treatment, and complications of perinatal infections with herpes 1, herpes 2, cytomegalovirus, Epstein-Barr virus, and varicella zoster Understand the epidemiology, pathogenesis, prevention, clinical manifestations, diagnostic criteria, treatment, and complications of perinatal infections with rubella Understand the epidemiology, pathogenesis, prevention, clinical manifestations, diagnostic criteria, treatment, and complications of perinatal infections with Treponema pallidum

 

 

http://emb.aap.org/courseprodv2/Index.asp[3/29/2012 2:47:53 PM]

 

  

Education Module Learner

Questions

June 04

Assessment Summary

CME Credit Expired

Page     1  2  3  4  5  6  7  8  9  10     Help  |  Table of Contents

Overview Assessment 04

An infant has a port-wine stain on the face. His parents are concerned about the appearance of the lesion and request advice about available treatments. Of the following, the modality that offers the BEST cosmetic palliation is

1

January 04

2

February 04

3

March 04

4

April 04

5

May 04

radiation therapy

6

June 04

systemic corticosteroid therapy

7

July 04

You selected

8

August 04

9

September 04

Port-wine stains (PWS) are permanent vascular malformations that are present at birth in 0.3% of infants. Lesions may occur anywhere on the body, but most often they are located on the face. Over time, PWS on the face darken, often develop vascular nodules that bleed, and may cause overgrowth of tissue. Several studies indicate, not surprisingly, that facial PWS frequently are a source of emotional distress.

10 October 04 November 04 December 12 04 11

NeoReviews Basic Self Assessment Return to NeoReviews.org

cryosurgery excision and grafting pulsed dye laser

, the correct answer is

.

Most experts agree that the treatment of choice for a patient who has significant facial PWS is therapy with a pulsed dye laser. The majority of children experience a reduction in the size of the PWS following treatment and a few will have complete resolution. Factors that affect the treatment outcome include the location and size of the lesion and the age at which therapy was begun. Treatment-related reductions in PWS size appear to be greatest when a lesion is located on the central forehead, followed in order by those on the peripheral face (eg, lateral forehead, cheeks, temples, jaws and chin), central face (eg, nose, upper lip, and fatty cheeks), and those that are mixed in location. Smaller lesions are more likely to diminish in size and become lighter after treatment than larger lesions. In one study, children who had lesions smaller than 20 cm2 had a mean reduction in PWS size of 60%, and 32% achieved complete resolution. In contrast, children who had larger PWS had a mean size reduction of only 41%, and none had complete resolution. Finally, early treatment appears to be associated with the best outcome. When laser treatment was initiated during the first year of life, one group of investigators documented a mean PWS size reduction of 65% and complete resolution in 32% of treated infants. In contrast, those who began treatment between 2 and 6 years of age had a mean PWS size reduction of 54%, and only 17% achieved complete clearance. Other treatment modalities, such as cryosurgery, radiation therapy, or excision and grafting, are not indicated for the child described in the vignette. Although systemic corticosteroids may be effective in the treatment of infants who have hemangiomas that threaten vital structures, they are of no value in the management of PWS. References Morelli JG, Weston WL, Huff JC, Yohn JJ. Initial lesion size as a predictive factor in determining the response of port-wine stains in children treated with the pulsed dye laser. Arch Pediatr Adolesc Med. 1995;149:1142-1144 Nguyen CM, Yohn JJ, Huff C, Weston WL, Morelli JG. Facial port wine stains in childhood: prediction of the rate of improvement as a function of the age of the

http://emb.aap.org/courseprodv2/Index.asp[3/29/2012 2:48:15 PM]

Education Module Learner

patient, size and location of the port wine stain and the number of treatments with the pulsed dye (585 nm) laser. Br J Dermatol. 1998;138:821-825 Strauss RP, Resnick SD. Pulsed dye laser therapy for port-wine stains in children: psychosocial and ethical issues. J Pediatr. 1993;122:505-510 Content Specification(s) Recognize, diagnose, and manage Port wine stain

 

 

http://emb.aap.org/courseprodv2/Index.asp[3/29/2012 2:48:15 PM]

 

  

Education Module Learner

Questions

June 04

Assessment Summary

CME Credit Expired

Page     1  2  3  4  5  6  7  8  9  10     Help  |  Table of Contents

Overview Assessment 04

A 3-day-old infant has been refusing to feed and is becoming increasingly lethargic. An evaluation for sepsis, including a complete blood count and cultures of blood, urine, and cerebrospinal fluid, is performed, and antibiotic therapy is initiated while awaiting the culture results.

1

January 04

Of the following, the MOST appropriate additional laboratory test to obtain now is

2

February 04

3

March 04

4

April 04

5

May 04

6

June 04

7

July 04

total and direct serum bilirubin concentrations

8

August 04

urine mucopolysaccharide concentration

9

September 04

plasma very long-chain fatty acids serum ammonia level stool porphyrin levels

You selected

, the correct answer is

.

10 October 04

The preferred response is 2.  serum ammonia level.

November 04 December 12 04

Acute illness in the neonate typically is characterized by nonspecific symptoms, regardless of the underlying etiology. Symptoms may include alterations in tone, suck, feeding, and respirations as well as lethargy. The refusal to feed and increasing lethargy described for the infant in the vignette should prompt an evaluation directed at identifying either an infectious or metabolic cause of the symptoms. Investigations should be conducted concurrently because delay in diagnosing an inborn error of metabolism can result in substantial neurologic damage that may be permanent.

11

NeoReviews Basic Self Assessment Return to NeoReviews.org

In particular, the deterioration of a previously well neonate suggests an intoxication type of metabolic disorder. These problems typically present with poor suck and feeding followed by increasing lethargy and coma. The most common ones are the organic acidemias, maple syrup urine disease, urea cycle defects, and nonketotic hyperglycinemia. Once a metabolic disorder is suspected, laboratory testing should be conducted immediately to aid in the precise diagnosis and to permit appropriate intervention. Ammonia and lactic acid levels, tests that are readily available in most hospital laboratories, should be measured immediately to guide the subsequent evaluation. Serum amino acid and urine organic acid analyses, which many hospitals refer to specialty laboratories, also should be initiated. Hyperammonemia with ketoacidosis is highly suggestive of an organic acidemia, and the precise diagnosis will be confirmed by analysis of organic acids. Some organic acidemias also result in granulocytopenia and thrombocytopenia, laboratory findings that frequently are mistaken for sepsis. This mistaken diagnosis can delay the further evaluation of affected infants whose clinical condition will continue to deteriorate despite broadspectrum antibiotic therapy. Alternatively, hyperammonemia with respiratory alkalosis is more suggestive of a urea cycle defect, a diagnosis that can be confirmed either by detection of a typical pattern of amino acid abnormalities or by demonstration of the specific enzymatic deficiency in a liver biopsy specimen. Stool porphyrin levels are useful in the diagnosis of the porphyrias, which are inherited defects of porphyrin metabolism. However, most of these disorders present later in life with cutaneous symptoms or abdominal pain. Determination of urine mucopolysaccharide concentrations is helpful in the initial evaluation of a child

http://emb.aap.org/courseprodv2/Index.asp[3/29/2012 2:48:33 PM]

Education Module Learner

suspected of having a mucopolysaccharidosis. These disorders typically present later in infancy with coarse facial features, organomegaly, and developmental delay. Definitive diagnosis following suggestive findings on the urine test requires demonstration of the specific enzymatic deficiency in peripheral blood leukocytes or cultured fibroblasts. Total and direct serum bilirubin concentrations may be obtained as part of the evaluation of a sick infant, but this result would not be specific enough to lead to the correct diagnosis. Very long-chain fatty acid levels are elevated in infants and children who have peroxisomal disorders. In the newborn period such conditions include Zellweger syndrome, but this disorder is characterized by dysmorphic facial features, hepatomegaly, and profound hypotonia that is evident from birth. References: Jones KL. Storage disorders. In: Smith's Recognizable Patterns of Human Malformation. 5th ed. Philadelphia, Pa: WB Saunders Co; 1997:450-470 Scriver CR, Beaudet AL, Sly WS, Valle D, eds. The Metabolic and Molecular Bases of Inherited Disease. 7th ed. New York, NY: McGraw-Hill Information Services Co; 1995 Content Specification(s): Recognize and diagnose the metabolic disorders that lead to coma PREP 2000    SA # 137

 

 

http://emb.aap.org/courseprodv2/Index.asp[3/29/2012 2:48:33 PM]

 

  

Education Module Learner

Questions

June 04

Assessment Summary

CME Credit Expired

Page     1  2  3  4  5  6  7  8  9  10     Help  |  Table of Contents

Overview Assessment 04

You are called to the newborn nursery to evaluate an infant whose serum glucose concentration is less than 30 mg/dL (1.67 mmol/L) at 2 hours after birth. His birthweight was 4,250 g. On physical examination, you note macroglossia, visceromegaly, glabellar nevus simplex (salmon patch), and a large umbilical hernia.

1

January 04

Of the following, the MOST likely cause of hypoglycemia in this infant is:

2

February 04

3

March 04

4

April 04

adrenal insufficiency

5

May 04

islet cell hyperplasia

6

June 04

7

July 04

8

August 04

9

September 04

10 October 04 November 04 December 12 04 11

NeoReviews Basic Self Assessment Return to NeoReviews.org

abnormal fatty acid oxidation

maternal diabetes pancreatic adenoma

You selected

, the correct answer is

.

The combination of a birthweight that is greater than the 95th percentile, macroglossia, visceromegaly, glabellar nevus simplex, and large umbilical hernia or omphalocele reported for the infant in the vignette strongly suggests the diagnosis of Beckwith-Wiedemann syndrome (BWS). In BWS, neonatal hypoglycemia is due to pancreatic islet cell hyperplasia that results in hyperinsulinemia. Hypoglycemia due to hyperinsulinemia in the newborn is reported most commonly in infants of diabetic mothers (IDMs). Although these babies are large for gestational age and have visceromegaly, there is usually a maternal history of diabetes, and the infants also may have polycythemia or hypocalcemia. IDMs generally do not have macroglossia or umbilical hernia/omphalocele. The hypoglycemia in both IDMs and infants who have BWS is due to islet cell hyperplasia. Hyperinsulinemia appearing in the first year of life typically is associated with islet cell hyperplasia and may be transmitted as an autosomal recessive trait. Pancreatic adenomas, another cause of hyperinsulin-emia, generally occur in older children and adolescents. Abnormal fatty acid oxidation, adrenal insufficiency, and nesidioblastosis do not present with macroglossia, macrosomia, or umbilical hernia/omphalocele. The primary goal of therapy for childhood hyperinsulinemia is to prevent the sequelae of hypoglycemia, including seizure disorder and mental retardation. For IDMs and those who have BWS, hyperinsulinemia usually is transient and requires the administration of intravenous dextrose to maintain normal serum glucose levels for only a few days. Children who have hyperinsulinemia due to pancreatic adenoma may require administration of glucocorticoids, diazoxide, dietary manipulation, and surgery in addition to glucose supplementation. BWS is an overgrowth syndrome associated with molecular aberrations within the chromosome 11p15 region that have not yet been characterized fully. Once the neonatal metabolic derangements are treated and morphologic concerns are addressed, the prognosis is excellent. References Haymond MW. Hypoglycemia in infancy and childhood. In: Rudolph AM, Hoffman JIE,

http://emb.aap.org/courseprodv2/Index.asp[3/29/2012 2:48:49 PM]

Education Module Learner

Rudolph CD, eds. Rudolph's Pediatrics. 20th ed. Stanford, Conn: Appleton & Lange; 1996:1832-1833 Wilson GN, Cooley WC. Beckwith-Wiedemann syndrome. In: Preventive Management of Children With Congenital Anomalies and Syndromes. Cambridge, United Kingdom: Cambridge University Press; 2000:273-276 Content Specification(s) Recognize the etiology, clinical manifestations, laboratory features, approach to therapy, and potential sequelae of neonatal hypoglycemia

 

 

http://emb.aap.org/courseprodv2/Index.asp[3/29/2012 2:48:49 PM]

 

  

Education Module Learner

Questions

June 04

Assessment Summary

CME Credit Expired

Page     1  2  3  4  5  6  7  8  9  10     Help  |  Table of Contents

Overview Assessment 04

An African-American newborn has pustules without surrounding erythema on the trunk and forehead and small (2 mm) hyperpigmented macules, some of which have peripheral scale. Of the following, the MOST appropriate management is to

1

January 04

2

February 04

administer acyclovir intravenously

3

March 04

administer cephalexin orally

4

April 04

advise no therapy

5

May 04

6

June 04

7

July 04

8

August 04

9

September 04

10 October 04 November 04 December 12 04 11

NeoReviews Basic Self Assessment Return to NeoReviews.org

apply mupirocin topically apply nystatin topically

You selected

, the correct answer is

.

One disorder that commonly affects African-American newborns is transient neonatal pustular melanosis, a condition of unknown cause that begins in utero. It is characterized by pustules and small (two to three mm) hyperpigmented macules that are present at birth. The macules have peripheral scale that represents the remnant of a pustule roof. Although the clinical diagnosis usually is straightforward, if there is uncertainty, a Gram or Wright stain of the pustule contents will reveal polymorphonuclear neutrophils without organisms. If indicated, a bacterial, viral, or fungal culture may be performed to exclude other causes. No treatment is required; pustules resolve within 5 days, with hyperpigmented macules persisting for up to 3 months. A number of other disorders can produce pustules or vesicles in the neonatal period. Cutaneous herpes simplex virus infection is characterized by the presence of clustered vesicles on an erythematous base. Due to the risk of associated central nervous system or disseminated infection, affected infants merit careful evaluation and treatment with acyclovir parenterally. Folliculitis due to Staphylococcus aureus results in pustules that have a surrounding rim of erythema. Depending on the extent of infection, topical mupirocin, oral cephalexin, or other appropriate antibiotic may be employed. Parenteral therapy is warranted if infants exhibit signs of bacterial sepsis. Cutaneous candidiasis causes diffuse scaling and erythema or erythematous papules and pustules. It may be present at birth (congenital) or appear days to weeks following delivery (acquired). Treatment with a topical antiyeast preparation (eg, nystatin) is indicated. References: Vasiloudes P, Morelli JG, Weston WL. A guide to rashes in newborns. Contemp Pediatr. 1997;14(6):156-167 Article Weston WL, Lane AT, Morelli JG. Skin diseases in newborns. In: Color Textbook of Pediatric Dermatology. 2nd ed. St. Louis, Mo: Mosby Year-Book; 1996:326-353 Content Specification(s): Understand how to recognize, diagnose and manage hyperpigmentation, including Café au lait spots, Peutz-Jeghers syndrome, giant hairy nevus, incontinentia pigmenti, and pigmented nevi

http://emb.aap.org/courseprodv2/Index.asp[3/29/2012 2:49:07 PM]

Education Module Learner

 

 

http://emb.aap.org/courseprodv2/Index.asp[3/29/2012 2:49:07 PM]

 

  

Education Module Learner

Questions

June 04

Assessment Summary

CME Credit Expired

Page     1  2  3  4  5  6  7  8  9  10     Help  |  Table of Contents

Overview Assessment 04

A 10-day-old infant who weighed 1,750 g at birth and whose gestational age was 34 weeks is jaundiced. His total serum bilirubin concentration is 10.0 mg/dL and the direct fraction is 0.8 mg/dL. He is receiving intermittent orogastric feeding of expressed human milk and supplemental parenteral nutrition.

1

January 04

Of the following, the MOST likely explanation for these findings is

2

February 04

3

March 04

4

April 04

5

May 04

6

June 04

7

July 04

physiologic jaundice

8

August 04

pyloric stenosis

9

September 04

10 October 04 November 04 December 12 04 11

NeoReviews Basic Self Assessment Return to NeoReviews.org

Crigler-Najjar syndrome jaundice due to parenteral nutrition neonatal hepatitis

You selected

, the correct answer is

.

The major steps of bilirubin metabolism include conjugation with glucuronide in the liver, secretion via bile into the intestine, conversion of conjugated bilirubin into urobilinoids, and fecal excretion. A small portion of bilirubin is hydrolyzed by betaglucuronidase and reabsorbed into the circulation, adding to the hepatic load of unconjugated bilirubin. Beta-glucuronidase activity in the intestine is high during fetal life, but decreases after enteral feedings are established when the gut becomes colonized with bacterial flora. The preterm infant described in the vignette has a serum total bilirubin concentration that is compatible with physiologic jaundice. A low direct bilirubin fraction means that there is no evidence of a conjugated hyperbilirubinemia and cholestasis associated with parenteral nutrition or neonatal hepatitis. The presence of some gastric feedings makes the increased enterohepatic circulation of bilirubin unlikely. In the term newborn, physiologic jaundice is characterized by a progressive increase in the serum concentration of unconjugated bilirubin from approximately 2.0 mg/dL in cord blood to a peak of about 6.0 mg/dL by the third day of life. The serum concentration then decreases rapidly to approximately 2.0 mg/dL by the fifth day of life and reaches the normal adult value of less than 1.0 mg/dL by the tenth day of life. In contrast, physiologic jaundice is more severe and prolonged in the preterm newborn. The peak serum concentration of unconjugated bilirubin may be as high as 12.0 mg/dL and may not decline to the normal level until the end of the first month of life. This delay in the clearance of bilirubin in the preterm neonate is attributed primarily to immaturity of glucuronyl transferase activity, the enzyme responsible for the conjugation of bilirubin in the liver. Crigler-Najjar syndrome is a rare and severe form of chronic nonhemolytic unconjugated hyperbilirubinemia. The disease is inherited as an autosomal recessive trait and in its homozygous form is characterized by complete absence of bilirubin glucuronide formation in the liver. The persistence of an unconjugated hyperbilirubinemia at concentrations exceeding 20.0 mg/dL beyond the first week of life in the absence of hemolysis is characteristic of the disease. Prolonged administration (³2 wk) of parenteral nutrition in the newborn may result in

http://emb.aap.org/courseprodv2/Index.asp[3/29/2012 2:49:25 PM]

Education Module Learner

conjugated hyperbilirubinemia. In these patients, liver biopsy shows evidence of hepatocellular injury, with swelling of hepatocytes and resultant cholestasis. Extremely low-birthweight neonates (q14) present with less severe physical and developmental disabilities. Partial trisomy for the distal segment (13q14->qter), however, is associated with severe mental deficiency and characteristic facies. References: Baraitser M, Winter RM. Color Atlas of Congenital Malformation Syndromes. London, England: Mosby-Wolfe; 1996 Jones KL. Smith's Recognizable Patterns of Human Malformation. 5th ed. Philadelphia, Pa: WB Saunders Co; 1997 Content Specification(s): Recognize the clinical features associated with autosomal recessive disorders. Recognize the clinical findings and chromosomal pattern in trisomy 13. Recognize the consequences of the amniotic band syndrome.

http://emb.aap.org/courseprodv2/Index.asp[3/29/2012 3:12:43 PM]

Education Module Learner

Recognize dysmorphic syndromes associated with hearing loss such as Waardenburg and Goldenhar syndromes. Recognize the clinical manifestations of anatomic abnormalities of the urinary tract in infants.

 

 

http://emb.aap.org/courseprodv2/Index.asp[3/29/2012 3:12:43 PM]

 

  

NeoReviews Plus '04

Page 1 of 1

Table 1

Close

http://emb.aap.org/courseprodv2/popup.asp?Nav=gotoMod&targetModule=p5801&Cours...

3/29/2012

Education Module Learner

Questions

November 04

Assessment Summary

CME Credit Expired

Page     1  2  3  4  5  6  7  8  9  10  11  12    

Overview

Help  |  Table of Contents

Assessment 04 1

January 04

2

February 04

3

March 04

4

April 04

5

May 04

6

June 04

7

July 04

8

August 04

9

September 04

10 October 04 November 04 December 12 04 11

NeoReviews Basic Self Assessment Return to NeoReviews.org

 

 

http://emb.aap.org/courseprodv2/Index.asp[3/29/2012 3:33:45 PM]

 

  

Education Module Learner

Questions

November 04

Assessment 04 January 04

2

February 04

3

March 04

4

April 04

5

May 04

6

June 04

7

July 04

8

August 04

9

September 04

CME Credit Expired

Page     1  2  3  4  5  6  7  8  9  10  11  12    

Overview

1

Assessment Summary

Help  |  Table of Contents

10 October 04 November 04 December 12 04 11

NeoReviews Basic Self Assessment Return to NeoReviews.org

 

 

http://emb.aap.org/courseprodv2/Index.asp[3/29/2012 3:14:36 PM]

 

  

Education Module Learner

Questions

December 04

Assessment Summary

CME Credit Expired

Page     1  2  3  4  5  6  7  8  9  10     Help  |  Table of Contents

Overview Assessment 04 1

January 04

2

February 04

3

March 04

4

April 04

5

May 04

6

June 04

7

July 04

8

August 04

9

September 04

10 October 04 November 04 December 12 04 11

NeoReviews Basic Self Assessment Return to NeoReviews.org

You are called to evaluate a 7-hour-old male infant, whose estimated gestational age is 26 weeks. He is receiving mechanical ventilation and has had an abrupt decrease in oxygen saturation (Sao2 of 68%). Prior to this event, he had been weaned to a peak inspiratory pressure of 17 cm H2O, positive end-expiratory pressure of 6 cm H2O, ventilator rate of 26 breaths/min, and Fio2 of 0.38 in the timecycled, pressure-limited mode on the ventilator. After ventilating with a manual resuscitator th rough the endotracheal tube, the oxygen saturation rises to 88%. An arterial blood gas reveals: pH of 7.09, Paco2 of 82 torr, Pao2 of 57 torr, and base excess of -4 mEq/L. A chest radiograph shows gross hyperexpansion, with coarse linear and cystlike radiolucencies throughout the lung. Of the following, the intervention that is MOST likely to improve the pulmonary function of this infant is extracorporeal membrane oxygenation high-frequency jet ventilation prone positioning systemic corticosteroids thoracentesis

You selected

, the correct answer is

.

Pulmonary interstitial emphysema is one of several air-block syndromes that complicate respiratory distress syndrome (RDS) in preterm infants; others include pneumothorax, pneumomediastinum, pneumopericardium, pneumoperitoneum, subcutaneous emphysema, and intravascular air. All of these syndromes begin with some degree of pulmonary interstitial emphysema. High intra-alveolar pressure causes rupture of air into the perivascular sheath that surrounds intra-alveolar capillaries and into the lymphatic tree within the interstitial tissues of the lung. The presence of air in these spaces, rather than in alveolar saccules and the tracheobronchial tree, is pulmonary interstitial emphysema. Of note, interstitial air usually does not collect in peribronchial spaces. Air then may dissect along the perivascular sheath toward the hilum of the lung where blebs collect at the reflection of the visceral pleura onto the parietal pleura. Air also may dissect to form blebs under the subpleural surface of the lung. These blebs of air may be reabsorbed or dissect into the pleural space to cause pneumothorax. If the blebs of air at the hilum are large or drain into the mediastinum, pneumomediastinum results. Air also may dissect into the peritoneal space (pneumoperitoneum) along the pulmonary ligament, intravascular space (air emboli), and subcutaneous spaces; these latter air-block syndromes usually occur with severe respiratory failure that requires high levels of mechanical ventilation support. Microscopic sections of lungs with pulmonary interstitial emphysema show an irregular pattern of lymphatic distention, with air and arterioles enveloped by air. Radiographs show linear and cystlike radiolucencies of variable size. The linear densities extend from the hilum to the periphery of the lung and have few branches; they must be differentiated from air bronchograms seen with RDS, which most often are located near the hilum of the lung and have many branches. The cystlike radiolucencies range from 1.0 to 4.0 mm in diameter and may be oval or lobulated; they may cause the chest radiograph to look "spongy". Air trapped within the interstitial spaces of the lung increases gas volume, which also causes the lung

http://emb.aap.org/courseprodv2/Index.asp[3/29/2012 3:22:02 PM]

Education Module Learner

fields to appear hyperexpanded. The physiologic consequences of pulmonary interstitial emphysema include loss of compliance, increased airway resistance, ventilation-perfusion mismatching, increased pulmonary vascular resistance, decreased venous return to the heart, and reduced cardiac output. Hypercarbia, hypoxemia, hypotension, and increased cerebral venous pressure then may occur. High-frequency jet ventilation is the intervention most likely to improve respiratory failure in the infant described in the vignette. High-frequency jet ventilation improves respiratory failure in preterm infants after air-leak syndromes such as pulmonary interstitial emphysema and pneumothorax have developed. Ventilation pressures were reduced and pulmonary interstitial emphysema resolved more rapidly in a randomized, multicenter trial of high-frequency jet ventilation after pulmonary interstitial pressure developed. High-frequency jet ventilation is not recommended for primary treatment of RDS because it may be associated with periventricular leukomalacia. High-frequency oscillatory ventilation is considered an alternative to high-frequency jet ventilation in preterm infants who have pulmonary interstitial emphysema. Extracorporeal membrane oxygenation is the prolonged use of cardiopulmonary bypass in neonates who have respiratory or cardiac failure and are failing to respond to maximal medical therapies. Because the risk of cerebral hemorrhage in infants who weigh less than 2 kg or are younger than 34 weeks' gestation at birth is high due to use of systemic heparin during cardiopulmonary bypass, extremely lowbirthweight infants are not candidates for this intervention. Prone positioning has not been proven to improve the outcome of extremely lowbirthweight infants who have pulmonary interstitial emphysema. Many clinicians position the infant with the involved lung down when unilateral pulmonary interstitial emphysema is present. Alternatively, unilateral mainstem intubation has been used in case studies for infants who have unilateral pulmonary interstitial emphysema. The use of systemic corticosteroids in extremely preterm infants who have respiratory failure generally is reserved for those who have severe chronic lung disease, hypotension unresponsive to vasopressors, or suspected vocal cord or tracheal edema at time of extubation or for infants enrolled in randomized, controlled trials. Although dexamethasone may reduce the risk of death or bronchopulmonary dysplasi a when administered early in the course of respiratory failure, the risk of adverse effects is high. The high risk for intestinal perforation, cerebral palsy, growth failure, hypertension, hyperglycemia, and adrenal suppression should be considered carefully when deciding to use systemic corticosteroids during the first postnatal weeks and months in preterm infants who have respiratory failure. Thoracentesis to decompress pulmonary interstitial emphysema without a large pneumothorax is unlikely to improve the outcome for the infant in the vignette. In fact, thoracentesis may cause pneumothorax and acute deterioration in cardiopulmonary status. References: Banks-Randall BA, Ballard RA. Bronchopulmonary dysplasia. In Tausch HW, Ballard RA, Gleason CA, eds. Avery's Diseases of the Newborn. 8th ed. Philadelphia, Pa: Elsevier Saunders; 2005:723-736 Carlo WA. Stark AR, Wright LL, et al. Minimal ventilation to prevent bronchopulmonary dysplasia in extremely-low-birth-weight infants. J Pediatr. 2002;141:370-375 Hansen TN, Corbet A, Gest AL, Moise AA. Principles of respiratory monitoring and therapy. In: Tausch HW, Ballard RA, Gleason CA, eds. Avery's Diseases of the Newborn. 8th ed. Philadelphia, Pa: Elsevier Saunders; 2005:660-662

http://emb.aap.org/courseprodv2/Index.asp[3/29/2012 3:22:02 PM]

Education Module Learner

Korones SB. Complications. In: Goldsmith JP, Karotkin EH, eds. Assisted ventilation of the neonate. 4th ed. Philadelphia, Pa: Saunders; 2003:362-364 Keszler M, Donn SM, Bucciarelli RL, et al. Multicenter controlled trial comparing high-frequency jet ventilation and conventional mechanical ventilation in newborn infants with pulmonary interstitial emphysema. J Pediatr. 1991;119:85-93 Wiswell TE, Graziani LJ, Kornhauser MS, et al. High-frequency jet ventilation in the early management of respiratory distress syndrome is associated with a greater risk of adverse outcomes. Pediatrics. 1996;98:1035-1043 Wright LL, Horbar JD, Gunkel H, et al. Evidence from multicenter networks on the current use and effectiveness of antenatal corticosteroids in low birth weight infants. Am J Obstet Gynecol. 1995;173:263-269 Content Specifications: Understand the indications for and techniques of high-frequency ventilation Understand the pathophysiology of air leaks Recognize the radiographic features of air leaks Understand how to prevent and manage air leaks

 

 

http://emb.aap.org/courseprodv2/Index.asp[3/29/2012 3:22:02 PM]

 

  

NeoReviews Plus '04

Page 1 of 1

Coarse linear and cystlike radiolucencies throughout the lung

Close

http://emb.aap.org/courseprodv2/popup.asp?Nav=gotoMod&targetModule=p6118&Cours...

3/29/2012

Education Module Learner

Questions

January 04

Asessment Summary

CME Credit Expired

Page     1  2  3  4  5  6  7  8  9  10     Help  |  Table of Contents

Overview Assessment 04 1

January 04

2

February 04

3

March 04

4

April 04

5

May 04

6

June 04

7

July 04

8

August 04

9

September 04

10 October 04 November 04 December 12 04 11

NeoReviews Basic Self Assessment Return to NeoReviews.org

A newborn has ambiguous genitalia characterized by posterior fusion of the labioscrotal folds and clitoromegaly. Findings include: chromosome analysis, a normal female 46,XX pattern; sodium, 127 mEq/L; potassium, 6.5 mEq/L; markedly increased levels of 17-hydroxyprogesterone and androstenedione. You diagnose the salt-wasting form of congenital adrenal hyperplasia (CAH) due to 21-hydroxylase deficiency. When discussing options for future pregnancies, the MOST appropriate statement to include is that CAH is identified best by measurement of amniotic fluid 17-hydroxyprogesterone prenatal diagnosis of CAH can be determined by molecular analysis of fetal DNA the fetal gender should be determined by ultrasonography because only females are at risk the mother should receive dexamethasone therapy throughout all future pregnancies there is a 50% risk for an affected child in each future pregnancy

You selected

, the correct answer is

.

Congenital adrenal hyperplasia (CAH) is an autosomal recessive disorder that results from deficiency of the enzyme 21-hydroxylase. The gene for this enzyme has been identified, and it is now possible to determine the exact molecular defect(s) in most families, which permits precise prenatal diagnosis by examination of fetal DNA obtained by chorionic villus sampling or amniocentesis. To facilitate such studies, the affected child and both parents should undergo genetic counseling and molecular genotyping of the 21-hydroxylase gene prior to initiating another pregnancy. In addition, couples at risk should be advised that early detection in all future pregnancies is necessary to prevent or minimize the virilizing effects on affected female fetuses. CAH occurs as two distinct phenotypes: a simple virilizing form (25% to 33% of patients) and a salt-wasting form (67% to 75% of patients). In both forms, the deficiency of 21-hydroxylase results in decreased cortisol synthesis by the adrenal gland and the increased production of cortisol precursors (eg, 17hydroxyprogesterone) and androgens, which do not require 21-hydroxylase for their synthesis. Because fetal genital development is regulated by adrenal steroid synthesis, the presence of elevated androgen levels in affected female fetuses results in variable degrees of virilization. In the salt-wasting form of the disease, renal salt wasting due to deficient aldosterone synthesis also occurs. The degree of virilization in affected females is not predictive of salt wasting, and even those who have mild virilization must be monitored for this possibility. However, the presence or absence of salt wasting usually is consistent in affected members of the same family. Thus, for the family described in the clinical vignette, any future affected infants also would be expected to have the salt-wasting form of CAH. The management goals for pregnancies at risk for CAH are to identify affected fetuses of both sexes and to minimize virilization in affected females. To achieve these goals, oral dexamethasone therapy must be initiated as soon as the pregnancy is detected. This semisynthetic steroid can cross the placenta and suppress the fetal adrenal gland in affected female fetuses, thereby minimizing the virilizing effects. Fetal cells then are obtained either by chorionic villus sampling at 10 weeks'

http://emb.aap.org/courseprodv2/Index.asp[3/29/2012 3:27:56 PM]

Education Module Learner

gestation or by amniocentesis at 15 to 18 weeks' gestation. DNA from these cells then is used to determine the fetal sex by chromosome analysis and for molecular testing (ie, 21-hydroxylase genotyping) to determine the disease status. If a male fetus is identified by the chromosome analysis, the dexamethasone can be discontinued. If the fetus is identified as female, the dexamethasone is discontinued only if results of the molecular analysis reveal that the fetus is unaffected. For pregnancies involving affected female fetuses, treatment is continued until delivery. All infants should be tested at birth to confirm the prenatal diagnosis by measuring 17-hydroxyprogesterone levels and repeating molecular analysis. Infants who were identified as affected by the prenatal testing also should have electrolyte levels measured on the first and second day of life to monitor for the salt-wasting form of CAH. Prior to the availability of molecular testing, determination of 17hydroxyprogesterone levels in amniotic fluid was used to identify fetuses who had CAH. Although 17-hydroxyprogesterone levels clearly are elevated in the severe saltwasting form of CAH, they may be normal in the simple virilizing form. Thus, this method lacks sensitivity and has been replaced by molecular testing for those families in which the mutation has been identified. Although ultrasonographic determination of fetal sex is possible, the prenatal management of CAH requires precise determination of the sex of the fetus, which is achieved best by fetal cell sampling and chromosome analysis. Because CAH is inherited as an autosomal recessive trait, the risk for an affected infant in each pregnancy is 25%. References: Karaviti LP, Mercado AB, Mercado MB, et al. Prenatal diagnosis/treatment in families at risk for infants with steroid 21-hydroxylase deficiency (congenital adrenal hyperplasia). J Steroid Biochem Mol Biol. 1992;41: 445-451 New MI, Crawford C, Wilson RC. Genetic disorders of the adrenal gland. In: Rimoin DL, Connor JM, Pyeritz RE, eds. Emery and Rimoin's Principles and Practice of Medical Genetics. 3rd ed. New York, NY: Churchill Livingstone; 1997:1441-1476 Content Specification(s) Know the etiology and diagnosis of an infant with ambiguous genitalia, including congenital adrenal hyperplasia

 

 

http://emb.aap.org/courseprodv2/Index.asp[3/29/2012 3:27:56 PM]

 

  

Education Module Learner

Questions

January 04

Asessment Summary

CME Credit Expired

Page     1  2  3  4  5  6  7  8  9  10     Help  |  Table of Contents

Overview

1

January 04

A 1-day-old infant appears dusky during feeding. Oxygen is administered via nasal cannula, and 2 hours later she develops tachypnea. Findings include: heart rate, 170 beats/min; respiratory rate, 80 breaths/min; right arm blood pressure, 48/30 mm Hg; right leg blood pressure, 52/32 mm Hg; and pulse oximetry, 90% on oxygen. You suspect hypoplastic left heart syndrome.

2

February 04

Of the following, the MOST likely findings include

3

March 04

4

April 04

5

May 04

6

June 04

7

July 04

8

August 04

9

September 04

Assessment 04

10 October 04 November 04 December 12 04 11

NeoReviews Basic Self Assessment Return to NeoReviews.org

continuous ductal murmur, bounding pulses continuous ductal murmur, poor peripheral pulses holosystolic murmur, poor peripheral pulses, quiet second heart sound no murmur, precordial hyperactivity, loud second heart sound no murmur, precordial hyperactivity, quiet second heart sound

You selected

, the correct answer is

.

Infants who have hypoplastic left heart (HLH) syndrome develop signs of shock, including poor pulses and metabolic acidosis, as systemic perfusion deteriorates. Ductal closure in these infants results in inadequate blood flow to the body from the functional single ventricle because the ductus arteriosus is the only path for blood to flow from the right ventricle to the body. Even if the ductus remains open, a drop in pulmonary vascular resistance "steals" blood to the pulmonary circulation, thereby depriving the systemic circulation of adequate perfusion. For this reason, mildly cyanotic infants who have HLH syndrome often deteriorate suddenly after oxygen is administered, because the oxygen acts as a pulmonary vasodilator. The most obvious physical findings in newborns who have HLH syndrome are a hyperdynamic precordium and a loud, even palpable single second heart sound (S2). The hyperactive precordium is due to the greatly enlarged right ventricle that is contracting against systemic pressure. The strikingly loud S2 is caused by the pulmonary artery functioning as a de facto aorta, pumping blood to the body through the ductus arteriosus. Therefore, the pulmonic closure sound is loud as a result of the high end systolic pressure found in the large pulmonary artery located just beneath the chest wall to the left of the sternum. A continuous murmur is not heard in infants who have HLH syndrome, even though the ductus arteriosus remains open. A murmur from a ductus is created by highvelocity flow from a higher pressure aorta into a lower pressure pulmonary artery. In patients who have HLH, the pulmonary artery pressure is equal to or exceeds that of the aorta. Because flow from the systemic right ventricle to this large hypertensive pulmonary artery is not obstructed or turbulent, there is no reason for a significant murmur to occur. A holosystolic murmur will be present only if there is very significant regurgitation of the tricuspid valve, which is unusual in affected infants. Diminished peripheral pulses are a sign of HLH syndrome, particularly as the ductus starts to close or as pulmonary vascular resistance falls. Both of these changes diminish the flow of systemic blood from the main pulmonary artery through the ductus to the systemic circulation. Maintaining ductal patency by administering an infusion of prostaglandin E1 often is not sufficient to provide adequate systemic blood flow in infants who have HLH

http://emb.aap.org/courseprodv2/Index.asp[3/29/2012 3:28:29 PM]

Education Module Learner

syndrome. Use of the so-called "chemical banding" of the pulmonary vasculature employs ventilatory strategies to increase pulmonary vascular resistance. Administration of room air while on the ventilator (or even Fio2 less than 21% using carbon dioxide in the ventilator circuit) and muscle relaxant drugs to prevent spontaneous hyperventilation with secondary pulmonary vasodilation often can reverse metabolic acidosis and re-establish good renal and systemic perfusion after prostaglandin E1 infusion has opened the ductus arteriosus. References: Victorica BE. Newborns with low systemic output. In: Gessner IH, Victorica BE, eds. Pediatric Cardiology: A Problem Oriented Approach. Philadelphia, Pa: WB Saunders Co; 1993:111-115 Wernovsky G, Bove EL. Single ventricle lesions: excessive pulmonary blood flow (congestive heart failure). In: Chang AC, Hanley FL, Wernovsky G, Wessel DL, eds. Pediatric Cardiac Intensive Care. Baltimore, Md: Williams & Wilkins; 1998:274  Content Specification(s): Recognize the clinical features of a neonate with a left-sided cardiac obstructive lesion

 

 

http://emb.aap.org/courseprodv2/Index.asp[3/29/2012 3:28:29 PM]

 

  

Education Module Learner

Questions

January 04

Asessment Summary

CME Credit Expired

Page     1  2  3  4  5  6  7  8  9  10     Help  |  Table of Contents

Overview

1

January 04

An 8-week-old child is being seen for a health supervision visit by his pediatrician. His mother reports that he is wheezing and having increasing difficulty while feeding. Physical examination reveals a raised, erythematous birthmark on the chest and biphasic stridor on auscultation.  The pediatrician calls a neonatologist to discuss the case.

2

February 04

Of the following, the MOST likely finding would be

3

March 04

4

April 04

5

May 04

6

June 04

7

July 04

obstructing adenoids on lateral neck radiography

8

August 04

unilateral air trapping on chest radiography

9

September 04

Assessment 04

10 October 04 November 04 December 12 04 11

NeoReviews Basic Self Assessment Return to NeoReviews.org

a prolapsing epiglottis on nasopharyngoscopy a unilateral subglottic mass on fluoroscopy left mainstem bronchomalacia on bronchoscopy

You selected

, the correct answer is

.

Biphasic stridor is produced by rapid turbulent flow of air through a narrowed segment of the airway. Stridor that is biphasic suggests an anatomic location at the glottic or subglottic level. Inspiratory stridor typically is produced by an obstructive lesion above the vocal cords, and expiratory stridor indicates an intrathoracic site of obstruction. Subglottic hemangioma is a benign vascular malformation that typically enlarges over the first few months of life. Patients often are asymptomatic or minimally symptomatic at birth, then develop biphasic stridor, as reported for the infant in the vignette. The natural history is gradual enlargement until 1 year of age followed by slow involution to complete resolution. Fluoroscopy most commonly reveals a unilateral subglottic mass. Fifty percent of affected children have an associated cutaneous hemangioma, such as the erythematous birthmark described in the vignette. Definitive diagnosis is made by microlaryngoscopy in the operating room. Carbon dioxide laser excision, systemic or intralesional steroids, and tracheotomy have been used for treatment. Inspiratory stridor in infancy most commonly is caused by laryngomalacia or prolapse of the supraglottic structures into the glottic introitus upon inspiration. Laryngomalacia can be diagnosed by nasopharyngoscopy. It usually is benign and resolves without intervention in 90% of patients by 1-1/2 years. Vocal cord paralysis is another common cause of inspiratory stridor that may have a mild expiratory component. Airway fluoroscopy or nasopharyngoscopy is diagnostic. Expiratory stridor most commonly is caused by tracheomalacia. This may present at birth and improve slowly over time; intervention rarely is required. Vascular compression may occur from an aberrant innominate artery that can compress the anterior trachea. Vascular rings (eg, double aortic arch) can compress both the trachea and esophagus. Retroesophageal subclavian compression typically causes dysphagia, but it also may result in respiratory symptoms. A diagnosis of tracheal anomaly and vascular lesion may be suggested by findings on chest radiography or airway fluoroscopy. Barium swallow demonstrates a typical indentation from posterior vascular compression in patients who have vascular rings or an aberrant subclavian artery, but it will not identify anterior compression from an aberrant

http://emb.aap.org/courseprodv2/Index.asp[3/29/2012 3:28:47 PM]

Education Module Learner

innominate artery. Flexible or rigid bronchoscopy can be used to confirm the degree of compression of the trachea from any cause. In patients who have suspected vascular airway compression, magnetic resonance imaging or computed tomography is used to define abnormal vascular anatomy. Obstructing adenoids can cause upper airway obstruction. Patients typically snore without accompanying stridor when tonsils and adenoids are hypertrophied. Although lateral neck radiography is a helpful diagnostic test for children who have adenoid hypertrophy, the clinical findings described in the vignette do not make this diagnosis likely. Congenital subglottic stenosis may be either membranous or cartilaginous. Stridor is typically present at birth and may worsen slightly over time. The diagnosis may be made by plain radiography, fluoroscopy, or endoscopy. Microlaryngoscopy in the operating room is the most accurate diagnostic method. The anomaly usually is symmetric and bilateral or circumferential. Unilateral air trapping on chest radiography is suggestive of unilateral compression or obstruction of the bronchus. In older infants, the most common cause is a foreign body. Bronchomalacia or other bronchial anomalies are diagnosed most commonly by bronchoscopy, but clinical findings would include expiratory wheezing rather than biphasic stridor. References: Brodsky L. Congenital stridor. Pediatr Rev. 1996;17: 408-411 Cotton RT, Reilly JT. Stridor and airway obstruction. In: Bluestone CD, Stool SE, Kenna MA, eds. Pediatric Otolaryngology. 3rd ed. Philadelphia, Pa: WB Saunders Co; 1996:1275-1287 Gregg CM, Wiatrak BJ, Koopmann CF Jr. Management options for infantile subglottic hemangioma. Am J Otolaryngol. 1995;16:409-414 Holinger LD. Etiology of stridor in the neonate, infant and child. Ann Otol Laryngol. 1980;89:397-400 Content specification(s): Know the various causes of stridor in the newborn

 

 

http://emb.aap.org/courseprodv2/Index.asp[3/29/2012 3:28:47 PM]

 

  

Education Module Learner

Questions

January 04

Asessment Summary

CME Credit Expired

Page     1  2  3  4  5  6  7  8  9  10     Help  |  Table of Contents

Overview Assessment 04

A 2-week-old neonate who was born at 32 weeks' gestation has recovered from respiratory distress syndrome. He has been tolerating increasing volumes of enteral feedings via gavage. Over the past several feedings, abdominal distension, gastric residuals, and stools that are positive for blood have been noted.

1

January 04

2

February 04

3

March 04

4

April 04

5

May 04

6

June 04

7

July 04

pneumatosis intestinalis

8

August 04

thickening of the bowel wall

9

September 04

10 October 04 November 04 December 12 04 11

NeoReviews Basic Self Assessment Return to NeoReviews.org

Of the following, the radiographic finding MOST supportive of the diagnosis of necrotizing enterocolitis is: absence of luminal bowel gas generalized bowel distension intraperitoneal fluid

You selected

, the correct answer is

.

Pneumatosis intestinalis (gas in the bowel wall) is the radiographic hallmark of necrotizing enterocolitis (NEC) and is used to confirm the diagnosis suggested by clinical symptoms and signs in the high-risk neonate described in the vignette. Pneumatosis intestinalis may be seen as a bubbly or foamy gas pattern or as a linear or crescent distribution of gas in the bowel wall. It may extend into the portal venous circulation and be visible as linear branching lucencies overlying the liver. Portal venous gas is associated with severe disease. The pattern of luminal bowel gas in infants who have NEC is variable and nonspecific. It is decreased in some, but this finding also can be seen in the absence of NEC in sick infants, particularly those receiving drugs for muscle paralysis during mechanical ventilation. Generalized bowel distension is an early radiographic sign of NEC in some infants, but it also is seen in infants who are receiving nasal continuous positive airway pressure. A single persistent dilated loop of bowel rather than generalized bowel distension is more characteristic of NEC. Multiple dilated loops of bowel, particularly when localized in the right lower quadrant, also are consistent with NEC. When intraperitoneal fluid is present, the gas-filled loops of bowel float centrally in the abdomen and are separated from each other in the fluid. Ultrasonography is more sensitive and specific than radiography for detecting intraperitoneal fluid. Fluid also may accumulate in the peritoneal cavity as a result of other from causes (eg, hypoalbuminemia). In infants who have NEC, loops of bowel may be separated from each other by bowel walls that have been thickened by edema and hemorrhage. However, detection of thickening of the bowel wall requires a somewhat subjective interpretation and is nonspecific for the diagnosis of NEC. NEC affects predominantly preterm infants, with a markedly increased incidence in infants at younger gestational ages. Abdominal distension and gastric residuals, such as described for the infant in the vignette, are signs of feeding intolerance. The presence of bloody stools suggests that there has been necrosis of intestinal epithelium. A prudent management approach would be to stop enteral feeding, initiate antibiotic therapy, and observe for symptoms (eg, lethargy, apnea, poor

http://emb.aap.org/courseprodv2/Index.asp[3/29/2012 3:29:10 PM]

Education Module Learner

perfusion) and signs (eg, abdominal tenderness, muscular guarding, erythema of abdominal wall) that would indicate more serious involvement. Serial laboratory tests are important to evaluate the progression of the disease. Leukopenia, leukocytosis, thrombocytopenia, anemia, coagulopathy, electrolyte imbalance, acidosis, hypoxia, or hypercapnia are indicative of worsening disease. Serial abdominal radiographs help to determine the need for surgical intervention. Evidence of intestinal perforation or worsening disease despite appropriate medical management or are indications for surgery in infants who have NEC. References: Crissinger KD. Necrotizing enterocolitis. In: Fanaroff AA, Martin RJ, eds. NeonatalPerinatal Medicine: Diseases of the Fetus and Infant. 6th ed. St Louis, Mo: Mosby-Year Book, Inc; 1997:1333-1337 Morrison SC, Jacobson JM. The radiology of necrotizing enterocolitis. Clin Perinatol. 1994;21:347-363. Content Specification(s):  Understand the clinical manifestations, laboratory and radiographic findings of NEC.

 

 

http://emb.aap.org/courseprodv2/Index.asp[3/29/2012 3:29:10 PM]

 

  

Education Module Learner

Questions

January 04

Asessment Summary

CME Credit Expired

Page     1  2  3  4  5  6  7  8  9  10     Help  |  Table of Contents

Overview Assessment 04 1

January 04

2

February 04

3

March 04

4

April 04

5

May 04

6

June 04

7

July 04

8

August 04

9

September 04

10 October 04 November 04 December 12 04 11

NeoReviews Basic Self Assessment Return to NeoReviews.org

You are examining a newborn in the nursery and palpate a large mass in the abdomen. Of the following, the MOST likely diagnosis is:

autosomal dominant polycystic kidney disease horseshoe kidney multicystic kidney dysplasia renal vein thrombosis Wilms tumor

You selected

, the correct answer is

.

A palpable abdominal mass in a newborn constitutes a medical emergency. The most common cause of an abdominal mass in the neonatal period is hydronephrosis. One common cause of hydronephrosis is multicystic kidney dysplasia (MKD). This congenital malformation is caused by obstruction to urine flow during in utero development, which results in abnormal parenchymal development. Renal ultrasonography reveals multiple cysts scattered throughout the renal parenchyma, with marked echogenicity of the parenchyma, indicating nonspecific renal dysplasia. The cysts are of varying size, often replacing normal renal tissue. Because the anomaly usually is unilateral, most children who have MKD have normal renal function. However, because at least 50% of patients have an abnormality of the contralateral kidney, voiding cystourethrography (VCUG) to evaluate for vesicoureteral reflux is essential. Other anomalies of the contralateral kidney include ureteropelvic junction obstruction (with hydronephrosis), duplicated ureters, renal dysplasia, and hydronephrosis. If MKD is bilateral, renal failure is assured. MKD is associated with other malformations, including CHARGE syndrome; Jeune syndrome; fetal alcohol syndrome; Marfan syndrome; Noonan syndrome; prune belly (Eagle-Barrett) syndrome; rubella syndrome; trisomies 8, 9, 13, 18, 21, and 22; tuberous sclerosis; and Zellweger syndrome. Additionally, MKD occurs in infants of diabetic mothers. Serum electrolyte concentrations must be obtained in any newborn who has MKD. In most instances, it is difficult to distinguish between hydronephrosis and MKD. Therefore, a renal scan (eg, DMSA scan) should be performed. If the patient has hydronephrosis, evidence of renal function will be apparent on the scan. However, function may not be demonstrated with MKD. If the contralateral kidney is large and the serum creatinine level is normal, the contralateral kidney probably is normal, but follow-up renal ultrasonography is required to demonstrate progressive function of the contralateral kidney and further atrophy of the affected kidney. Repeat ultrasonography and measurement of serum creatinine levels annually are appropriate follow-up. Treatment neither is required nor available for unilateral MKD. Most affected kidneys eventually atrophy; hence, prior recommendations to perform nephrectomy in anticipation of possible malignant transformation no longer are advocated. However, annual renal ultrasonography should be performed to assess the size of the affected kidney.

http://emb.aap.org/courseprodv2/Index.asp[3/29/2012 3:29:28 PM]

Education Module Learner

Autosomal dominant polycystic kidney disease (ADPKD) is a very common cause of renal failure in adults. However, because it is a very slowly progressive disease, it rarely is seen in newborns. ADPKD typically presents with hematuria, urinary tract infection, and hypertension. Although horseshoe kidneys are infrequently observed in newborns, they may present with urinary tract infections, hematuria, or abdominal mass. Nephroblastoma is a general term used to describe a tumor of the kidney. The most common cause in early childhood is Wilms tumor, which presents as an abdominal or flank mass and may be heralded by hematuria and rarely, hypertension. It can be associated with other congenital anomalies, including genitourinary anomalies, hemihypertrophy, aniridia, gastrointestinal anomalies, polydactyly, and hydrocephalus. Although this is the most common solid tumor in childhood, it rarely presents during the newborn period. The typical presentation of renal vein thrombosis in a child is gross hematuria and an abdominal mass. Although rare, it often results in complete loss of function of the affected kidney. References: Atiyeh B, Husmann D, Baum M. Contralateral renal abnormalities in multicysticdysplastic kidney disease. J Pediatr. 1992;121:65-67 Kleiner B, Filly RA, Mack L, Callen PW. Multicystic dysplastic kidney: observations of contralateral disease in the fetal population. Radiology. 1986;161:27-29 Pathak IG, Williams DI. Multicystic and cystic dysplastic kidneys. Br J Urol. 1964;36:318-331 Content Specification(s): Recognize the clinical and laboratory features of common neonatal malignancies, including teratomas, hemangiomas, neuroblastoma, Wilms tumor, retinoblastoma.

 

 

http://emb.aap.org/courseprodv2/Index.asp[3/29/2012 3:29:28 PM]

 

  

NeoReviews Plus '04

Page 1 of 1

Multicystic Kidney Dysplasia

Courtesy of S Dabbaugh

Multiple cysts scattered throughout the kidney are characteristic of multicystic dysplasia.

Close

http://emb.aap.org/courseprodv2/popup.asp?Nav=gotoMod&targetModule=p3359&Cours...

3/29/2012

Education Module Learner

Questions

January 04

Asessment Summary

CME Credit Expired

Page     1  2  3  4  5  6  7  8  9  10     Help  |  Table of Contents

Overview Assessment 04 1

January 04

2

February 04

3

March 04

4

April 04

5

May 04

6

June 04

7

July 04

8

August 04

9

September 04

10 October 04 November 04 December 12 04 11

NeoReviews Basic Self Assessment Return to NeoReviews.org

You are discussing the risks of retinopathy of prematurity (ROP) with the parents of an infant who was born at 30 weeks' gestation. The child had moderately severe respiratory distress syndrome that required surfactant therapy, mechanical ventilation for 5 days, and supplemental oxygen for 2 weeks. Of the following, the statement that you are MOST likely to include in your discussion of ROP is: an ophthalmologic evaluation for ROP should be performed when the child is 4 to 6 weeks old birthweight is not a predictor of ROP boys are more likely than girls to develop ROP children who have ROP that regresses are unlikely to have visual impairment the incidence of ROP is higher in African-American infants

You selected

, the correct answer is

.

Retinopathy of prematurity (ROP) is a disorder of developing retinal blood vessels in preterm infants. The retinal blood vessels grow outward from the optic disc to the ora serrata during normal development. This centrifugal growth of the retinal blood vessels begins at 15 to 18 weeks of gestational age and is complete by about 44 weeks. Injury to the immature retinal blood vessels from factors such as hyperoxia, shock, asphyxia, hypothermia, and exposure to light can arrest the normal development of the retinal vasculature and result in ROP. Typically, ROP is visible in the retina only rarely before 4 to 6 weeks after birth. The rate of progression of the disease varies according to the gestational age. The peak incidence occurs at 34 to 42 weeks of gestational age. Therefore, a preterm infant whose birthweight was less than

Suggest Documents